76
1 Career Advisors FAQ List For questions regarding career counseling, please contact SOM Career Advising at [email protected] Updated 11/2019

Career Advisors FAQ List - education.uwmedicine.org...Career Advisors FAQ List For questions regarding career counseling, please contact SOM Career Advising at [email protected] Updated

  • Upload
    others

  • View
    2

  • Download
    0

Embed Size (px)

Citation preview

Page 1: Career Advisors FAQ List - education.uwmedicine.org...Career Advisors FAQ List For questions regarding career counseling, please contact SOM Career Advising at medadv@uw.edu Updated

1

Career AdvisorsFAQ List

For questions regarding career counseling, please contact SOM Career Advising at [email protected]

Updated 11/2019

Page 2: Career Advisors FAQ List - education.uwmedicine.org...Career Advisors FAQ List For questions regarding career counseling, please contact SOM Career Advising at medadv@uw.edu Updated

2

Anesthesiology

Dermatology

Emergency Medicine

Family Medicine

Medicine

Neurological Surgery

Neurology

Obstetrics-Gynecology

Ophthalmology

Orthopaedics

Otolaryngology

Pathology

Pediatrics

Physical Medicine and Rehabilitation

Plastic Surgery

Psychiatry

Radiation Oncology

Radiology

Surgery

UrologySTUDENT INTEREST GROUP DIRECTORY WEBPAGE

Dr. Tomoko Sairenji

Table of Contents

Dr. Ralph Ermoian

53

47

57

33

40

44

Dr. Jeffrey Friedrich

Drs. Mindy Loveless & Ny-Ying Lam

Dr. Kristine Calhoun

60

64

68

Dr. Anna Borisovskaya

22

72

25

29

50

Department Pages

7

10

3

Department Career AdvisorsDr. Michael Hall

Drs. Joshua Jauregui & Jared Strote

Dr. Jay Vary

14

Dr. Judith Hagedorn

Drs. Jonathan Medverd & Gautham Reddy

Drs. Mara Rendi & Jose Mantilla

76

Drs. Doug Paauw & Jenny Wright

Dr. Richard Ellenbogen

Dr. Eric Kraus

Dr. Vicki Mendiratta

Dr. Courtney Francis

Dr. Lisa Taitsman

Dr. Ian Humphreys

Dr. Michelle Terry

18

Page 3: Career Advisors FAQ List - education.uwmedicine.org...Career Advisors FAQ List For questions regarding career counseling, please contact SOM Career Advising at medadv@uw.edu Updated

3

Career Advisor

Dr. Michael Hall

Departmental Advisors

Drs. Murali Sivarajan, Wil Van Cleve, Melnanie Liu, Renata Ferreira, Andrew Pittaway, Steve Vanhoy, Bukky Ojo, Karen Souter

AnesthesiologyPreferred Method of Contact Contact Information

Email [email protected]

Anesthesiology Interest Group (AIG UWSOM)Dr. Erica Holand

[email protected]

Advisors Assigned by Department Departmental Career Advising Website

Yes N/A

Student Interest Group (S.I.G.) S.I.G. Contact Information

Frequently Asked QuestionsWhich clerkships are required for students to match in Anesthesiology? Clerkship in Pain Medicine recommended for those students with long-term career goal of pain medicine practice. Although not required, an anesthesiology clerkship is strongly recommended. The 4-week anesthesiology clerkship (Advanced Anesthesiology) is strongly preferred to the 2-week clerkship (Basic Anesthesiology):

-- 4 week clerkshipsANEST 650 – Seattle, WA (HMC)ANEST 670 – Bozeman, MTANEST 681 – Seattle, WA (UWMC)ANEST 682 – Seattle, WA (HMC)ANEST 683 – Seattle, WA (VAMC)ANEST 684 – Spokane, WA (Sacred Heart)ANEST 685 – Spokane, WA (Deaconnes)ANEST 688 – Cheyenne, WY

-- 2 week clerkshipsANEST 663 – Cheyenne, WY ANEST 665 – Anchorage, AKANEST 667 – Billings, MTANEST 668 – Missoula, MTANEST 669 – Bozeman, MTANEST 671 – Boise, ID

-- 2 week clerkshipsANEST 675 – Spokane, WA (Deaconness)ANEST 667 – Seattle, WA (Swedish)

Page 4: Career Advisors FAQ List - education.uwmedicine.org...Career Advisors FAQ List For questions regarding career counseling, please contact SOM Career Advising at medadv@uw.edu Updated

4

Do you recommend that students take an elective in Anesthesiology at another institution?Away electives are helpful for students interested in residency at a specific institution, or if they are unable to schedule a clerkship at a UW Medicine or WWAMI site in a timely fashion.

Which Advanced Patient Care clerkships help students prepare for residency in Anesthesiology?

Advanced Patient Care: Any APC in Anesthesia including pain managementPediatric Emergency MedicineCritical Care Medicine (non-subI)Critical Care APC in Medicine, or Subinternship in Critical Care Medicine taken for APC creditOutpatient Cardiology

Which UWSOM Sub-Is help students prepare for residency in Anesthesiology? General Surgery, Surgical Intensive Care Unit (Surgical ICU), Subinternship in Critical Care Medicine (Medical ICU), Subinternship in Neurocritical Care Medicine (Neurosurgical ICU, conjoint Med/Anesthesia), Cardiology Subinternship.

Are there any electives outside this specialty that you recommend to students interested in matching into Anesthesiology?

Cardiology or Clinical Respiratory Disease (Pulmonary); could also take additional critical care APC's or subinternships for elective credit.

Do you have any additional recommendations for students who are committed to matching in this specialty?

Because anesthesiologists practice in settings of the operating room, emergency room, intensive care unit, and pain clinic, students rotating through these clinical settings on non-anesthesiology clerkships should seek advice and clinical experience from anesthesiologists who may be working in these settings. For example, when on the required general surgery or obstetrics rotations, befriend the anesthesiologist, inquire about his/her job, and ask to participate in clinical care (as your time and other responsibilities allow). Research can be helpful, both to your own career exploration and your residency application, so seek such opportunities when interested.

Please list the most common mistakes students make when they apply to your specialty:

Students may not consider anesthesiology until late in their clinical clerkship years, which can make scheduling an anesthesiology clerkship difficult. See suggestion above about exploring anesthesiologists roles while participating in required, non-anesthesiology clerkships. Complete an anesthesiology clerkship no later than September of the MS4 year.

Page 5: Career Advisors FAQ List - education.uwmedicine.org...Career Advisors FAQ List For questions regarding career counseling, please contact SOM Career Advising at medadv@uw.edu Updated

5

5 4 3 2 1Highly Important Less Important

●●

If a student is strongly interested in Anesthesiology, please rate how important each of the following activities and performance factors are for a successful residency match.

Achieving 4 or more "Honors" in required clerkship rotations

Achieving "Honors" in this specialty's clerkship rotation

Doing an away rotation in this specialty ●Doing a summer research project in this specialty ●

Doing a preceptorship in this specialtyDoing a UWSoM Explore & Focus Sub-I in this specialty ●

3 letters of recommendation from physicians in this specialty

Express a strong interest in academic medicine ●

1 letter of reference from someone in this specialty1 letter of recommendation from a department chair in this specialty

Having a Step I USMLE score above 235Election to AOA ●

Having a published abstract or poster presentation ●Having at least one published paper in a peer reviewed journal

GHIP or other international experience ●Having an interest in practicing in a rural or underserved community ●

Having a Master's Degree (eg.MPH) ●Being bilingual ●

Participating in community service

Doing RUOP or other experience in underserved communities ●Election to Gold Humanism Honor Society (GHHS) ●

Page 6: Career Advisors FAQ List - education.uwmedicine.org...Career Advisors FAQ List For questions regarding career counseling, please contact SOM Career Advising at medadv@uw.edu Updated

6

Length of Residency Training

4 years

Rate Level of Competitiveness of Specialty

Medium

Mean # of Programs Applied to in this Specialty

35.6

National Characteristics of Entering Anesthesiology ResidentsCategorical vs. Advanced Program Median Salaries

# of U.S. M.D. Applicants in this Specialty # of Positions Offered

2,004 1,840

For more information, please visit the AAMC's Careers in Medicine: Anesthesiology website.

Both availableClinical Practice starting salary: $371,000Academic Med. Assistant Prof: $326,000Academic Med. Full Prof: $350,000

Mean USMLE Step 1 Score Mean USMLE Step 2 CK Score

232 244

Return to Table of Contents

At what point in the interview season should a student become concerned about their application to Anesthesiology residency programs?

Concern is raised if 10 interviews are not scheduled by November 1.

If a student is concerned about their application status, what should they do?

Contact the residency program(s) and his/her career advisor.

Residency Application Process - Frequently Asked QuestionsHow many programs should a student apply to in order to be successful in matching in Anesthesiology?

30-40 for "average applicant"

Recognizing that it can vary from year to year, nationally, when is the typical interview season for Anesthesiology?

November to Early February

Page 7: Career Advisors FAQ List - education.uwmedicine.org...Career Advisors FAQ List For questions regarding career counseling, please contact SOM Career Advising at medadv@uw.edu Updated

7

Career Advisor

Dr. Jay Vary

Departmental AdvisorsDrs. Andrea Kalus, Michi Shinohara, Katie DeNiro, April Schachtel, Roy Colven

DermatologyPreferred Method of Contact Contact Information

Email [email protected]

Which Advanced Patient Care clerkships help students prepare for residency in Dermatology? Most students will take a Dermatology APC (MEDECK 604)

Dermatology Interest Group This S.I.G. has a virtual presence at UWSOM.

Advisors Assigned by Department Departmental Career Advising Website

No https://derm.uw.edu/education/medical-students

Student Interest Group (S.I.G.) S.I.G. Contact Information

Do you recommend that students take an elective in Dermatology at another institution?Yes

Frequently Asked QuestionsWhich clerkships are required for students to match in Dermatology? Dermatology - 4 week (MEDECK 604 or CONJ 629)

Which UWSOM Sub-Is help students prepare for residency in Dermatology? Ward or Hospital Medicine, Hospitalist and Oncology, Subinternship (MEDECK 684), Oncology Subinternship, Clinical Burn Care, Plastic Surgery, Pediatrics (known as Advanced Pediatric Clerkship with APC-S designation or Advanced Pediatric Subinternship).

Are there any electives outside this specialty that you recommend to students interested in matching into Dermatology? Infectious disease, rheumatology, pediatric dermatology, pathology, ENT

Please list the most common mistakes students make when they apply to your specialty:

Not applying broadly enough. Not having multiple people read your personal statement. No Plan B; you will be asked about this during interviews.Deciding late to apply to dermatology can be stressful.Get letters of recommendation from at least 2 dermatologists, preferably from academic people who will more likely know colleagues are other training programs.

Do you have any additional recommendations for students who are committed to matching in this specialty?This is a competitive specialty, so best to have realistic expectations. Also, it's much harder to match if you are geographically restricted; it's important to apply broadly. Lastly, know your back up plan and discuss this with your advisor.

Page 8: Career Advisors FAQ List - education.uwmedicine.org...Career Advisors FAQ List For questions regarding career counseling, please contact SOM Career Advising at medadv@uw.edu Updated

8

5 4 3 2 1Highly Important Less Important

●●

● ●

If a student is strongly interested in Dermatology, please rate how important each of the following activities and performance factors are for a successful residency match.

Achieving 4 or more "Honors" in required clerkship rotations

Achieving "Honors" in this specialty's clerkship rotation

Doing an away rotation in this specialtyDoing a summer research project in this specialty ●

Doing a preceptorship in this specialty

Doing a UWSOM Explore & Focus Sub-I in this specialty ●

3 letters of recommendation from physicians in this specialty

Express a strong interest in academic medicine ●

1 letter of reference from someone in this specialty1 letter of recommendation from a department chair in this specialty ●

Having a Step I USMLE score above 235Election to AOA ●

Having a published abstract or poster presentation ●Having at least one published paper in a peer reviewed journal

GHIP or other international experience ●Having an interest in practicing in a rural or underserved community

Having a Master's Degree (eg.MPH) ●Being bilingual

Participating in community service

Doing a RUOP or other experience in an underserved communityElection to Gold Humanism Honor Society (GHHS)

Page 9: Career Advisors FAQ List - education.uwmedicine.org...Career Advisors FAQ List For questions regarding career counseling, please contact SOM Career Advising at medadv@uw.edu Updated

9

Length of Residency Training

4 years (including the PGY-1 year)

Rate Level of Competitiveness of Specialty

High

Mean # of Programs Applied to in this Specialty

59.3

National Characteristics of Entering Dermatology ResidentsCategorical vs. Advanced Program Median Salaries

# of U.S. M.D. Applicants in this Specialty # of Positions Available

919 ~450

For more information, please visit the AAMC's Careers in Medicine: Dermatology website.

Both availableClinical Practice starting salary: $425,500Academic Med. Assistant Prof: $291,000Academic Med. Full Prof: $353,000

Mean USMLE Step 1 Score Mean USMLE Step 2 CK Score

249 256

Return to Table of Contents

At what point in the interview season should a student become concerned about their application to Dermatology residency programs?

By December 15th if they don't have at least 5 interviews.

If a student is concerned about their application status, what should they do?

Best to contact Jay Vary AND the dermatologist(s) with whom you worked most closely.

Residency Application Process - Frequently Asked QuestionsHow many programs should a student apply to in order to be successful in matching in Dermatology?

At least 30.

Recognizing that it can vary from year to year, nationally, when is the typical interview season for Dermatology?

December and January

Page 10: Career Advisors FAQ List - education.uwmedicine.org...Career Advisors FAQ List For questions regarding career counseling, please contact SOM Career Advising at medadv@uw.edu Updated

10

Career Advisor

Drs. Joshua Jauregui and Jared Strote

Departmental Advisors

Drs. Adeyinka Adedipe, Annie Chipman, Doug Franzen, Dan Henning, Jon Ilgen, Joshua Jauregui, Ross Kessler, Jared Strote, David Townes, Jessica Wall

Emergency MedicinePreferred Method of Contact Contact Information

Email cc advisor: Alexis Rush on emails to Drs. Jauregui and Strote at [email protected]; [email protected]; [email protected]

Which Advanced Patient Care clerkships help students prepare for residency in Emergency Medicine?

Pediatric Emergency Medicine, Ortho/Sports Medicine, Outpatient Cardiology, Opioid Use Disorder, many others would be relevant - choose by interest.

Emergency Medicine Interest Group (EMIG)Dr. Joshua [email protected]

Advisors Assigned by Department Departmental Career Advising Website

Yes https://em.uw.edu/education/em-career-info

Student Interest Group (S.I.G.) S.I.G. Contact Information

Are there any electives outside this specialty that you recommend to students interested in matching into Emergency Medicine?

Those that most interest the student. Most all clerkships are pertinent to the broad specialty of Emergency Medicine.

Frequently Asked QuestionsWhich clerkships are required for students to match in Emergency Medicine? All core required clerkships. Emergency Medicine required clerkship at HMC/UWMC

Do you recommend that students take an elective in Emergency Medicine at another institution?

Yes, must do one away elective/subinternship at another institution.

Which UWSOM Sub-Is help students prepare for residency in Emergency Medicine?Subinternship in Critical Care Medicine (Medical ICU), Surgical Intensive Care Unit (Surgical ICU), Cardiology Subinternship, Clinical Burn Care, Family Medicine, Ward or Hospital Medicine.

Page 11: Career Advisors FAQ List - education.uwmedicine.org...Career Advisors FAQ List For questions regarding career counseling, please contact SOM Career Advising at medadv@uw.edu Updated

11

Do you have any additional recommendations for students who are committed to matching in this specialty?Take EM at HMC/UWMC in the spring/summer of your 4th year, and plan to have time for an away elective in EM in later summer/early fall. It is helpful to do your away rotation in this timing so that you can have a letter from your away rotation early in the application process.

Programs like to see that you have committed yourself to something outside of your studies during medical school. This can be service, research, or other interests. A pattern of dedication to an area of interest is of benefit to your application.

If you think you may be interested in EM as a career, we recommend getting involved with EMIG early on, as a member and possibly as a leader. Preceptorships and shadowing opportunities in EM during Foundations also provide you early exposure to the field and can help guide your career exploration.

Please list the most common mistakes students make when they apply to your specialty:Not applying to enough programs, or applying only to highly competitive programs or highly competitive regions. To avoid this, it is helpful to sit down with your career advisor and take a full assessment of your academic profile and the type and number of programs you should apply to.Not planning ahead to make time for an away rotation, or not being flexible with the timing for an away rotation.Not communicating regularly with career advisors, especially with concerns. Advisors are student advocates, and can be most helpful when students share information. Not being open to a parallel application plan - EM is a competitive speciality and some students must have a dual-application plan in a less-competitive specialty.

Page 12: Career Advisors FAQ List - education.uwmedicine.org...Career Advisors FAQ List For questions regarding career counseling, please contact SOM Career Advising at medadv@uw.edu Updated

12

5 4 3 2 1Highly Important Less Important

●●

●●

If a student is strongly interested in Emergency Medicine, please rate how important each of the following activities and performance factors are for a successful residency match.

Achieving 4 or more "Honors" in required clerkship rotations ●

Achieving "Honors" in this specialty's clerkship rotation

Doing an away rotation in this specialtyDoing a summer research project in this specialty

Doing a preceptorship in this specialty

Doing a UWSOM Explore & Focus Sub-I in this specialty(the required rotation counts as your home subinternship in EM) ●

3 letters of recommendation from physicians in this specialty ●

Express a strong interest in academic medicine

1 letter of reference from someone in this specialty1 letter of recommendation from a department chair in this specialty ●

Having a Step I USMLE score above 235 ●Election to AOA

Having a published abstract or poster presentationHaving at least one published paper in a peer reviewed journal

GHIP or other international experience ●Having an interest in practicing in a rural or underserved community ●

Having a Master's Degree (eg.MPH) ●Being bilingual

Participating in community service ●

Doing a RUOP or other experience in an underserved community ●Election to Gold Humanism Honor Society (GHHS) ●

Page 13: Career Advisors FAQ List - education.uwmedicine.org...Career Advisors FAQ List For questions regarding career counseling, please contact SOM Career Advising at medadv@uw.edu Updated

13

Length of Residency Training

There are 3-year and 4-year programs

Rate Level of Competitiveness of Specialty

Medium to High

Mean # of Programs Applied to in this Specialty

50.3

National Characteristics of Entering Emergency Medicine ResidentsCategorical vs. Advanced Program Median Salaries

# of U.S. M.D. Applicants in this Specialty # of Positions Available

3,258 2,278

For more information, please visit the AAMC's Careers in Medicine: Emergency Medicine website.

CategoricalClinical Practice starting salary: $358,000Academic Med. Assistant Prof: $285,000Academic Med. Full Prof: $321,000

Mean USMLE Step 1 Score Mean USMLE Step 2 CK Score

233 247

Return to Table of Contents

At what point in the interview season should a student become concerned about their application to Emergency Medicine residency programs?

By mid to late October, if a student has fewer than 8-10 interviews, they should contact their advisor.

If a student is concerned about their application status, what should they do?

Contact EM faculty career advisor.

Residency Application Process - Frequently Asked QuestionsHow many programs should a student apply to in order to be successful in matching in Emergency Medicine?

Varies based on your academic profile, average 40-60

Recognizing that it can vary from year to year, nationally, when is the typical interview season for Emergency Medicine?

Some interviews start in October, although most from November through early January. It is helpful to have time off somewhere in Nov-Jan. Applicants with fewer Honors grades or lower Step scores may benefit from scheduling time off on the later side, in December and January, to potentially pick up additional interviews as cancellations occur. Having a bit of flexibility throughout the Nov-Jan season is helpful.

Page 14: Career Advisors FAQ List - education.uwmedicine.org...Career Advisors FAQ List For questions regarding career counseling, please contact SOM Career Advising at medadv@uw.edu Updated

14

Career Advisor

Dr. Tomoko Sairenji

Departmental Advisors

Drs. Jeanne Cawse-Lucas, Tomoko Sairenji, and Ivan Henson (first contact should go through Ivan at [email protected])

Family MedicinePreferred Method of Contact Contact Information

Email [email protected]

Do you recommend that students take an elective in Family Medicine at another institution?Not necessary unless there is a program or region outside of WWAMI that you are very interested in.

Family Medicine Interest Group (FMIG)Dr. Tomoko Sairenji

[email protected]

Advisors Assigned by Department Departmental Career Advising Website

Yes

https://depts.washington.edu/fammed/education/advising/ (for general info)

https://depts.washington.edu/fammed/education/advising/apply/ (For advice on applying to residency)

Student Interest Group (S.I.G.) S.I.G. Contact Information

Which Advanced Patient Care clerkships help students prepare for residency in Family Medicine?

Second FM sub-I sometimes recommended. Rural (RUOP/WRITE site-ffiliated) APC if interested in rural medicine with little experience in this setting. Otherwise, choose by interest. Consider the following:Family Medicine APCs, Chronic Care/Pain Management (Anesthesia), Medical Consultation, Palliative Care, Opioid Use Disorder, Dermatology, Advanced Inpatient Geriatrics or Advanced Outpatient Geriatrics (Medicine), Outpatient Cardiology, Ortho/Sports Medicine, Psychiatry, OB/GYN.

Which UWSOM Sub-Is help students prepare for residency in Family Medicine?

Frequently Asked QuestionsWhich clerkships are required for students to match in Family Medicine?

One FM Sub-I recommended

FM sub-I is recommended. It helps indicate interest in the field in addition to building clinical skills. Some students do two, but that is not necessary. It is possible to do a second sub-I in IM (Hospitalist, Ward Medicine, Critical Care) if desired.

Page 15: Career Advisors FAQ List - education.uwmedicine.org...Career Advisors FAQ List For questions regarding career counseling, please contact SOM Career Advising at medadv@uw.edu Updated

15

Are there any electives outside this specialty that you recommend to students interested in matching into Family Medicine?

Students should pursue any electives that will help them feel prepared for an internship in FM. Some electives that are popular include radiology, sports medicine, cardiology, and dermatology. Many students also pursue electives that will help them prepare for a particular interest within FM training (for example, OB/GYN, psych, pain).

Do you have any additional recommendations for students who are committed to matching in this specialty?

Talk to a FM career advisor early, especially if you think you might have yellow/red flags (as soon as you have identified there may be a problem).

Please list the most common mistakes students make when they apply to your specialty:Be honest and forthcoming with your FM career advisors, especially about concerns or red flags. Keep them updated during the application process. If you are dual applying, it is very important to tell your FM career advisor; this can change your application strategy. We are not involved in the ranking process of any of the WWAMI Network FM Residency programs, and we would like to you to feel safe about discussing dual application. Ask for help when you need it. Work with a FM advisor to determine the right number of programs. Have one strong letter in FM. There is no need for a chair's letter. Other strong letters from any specialty or those from faculty who have worked with you closely and observed you clinically. Make sure you contact Ivan ([email protected]) to be on the advising listserv.

Page 16: Career Advisors FAQ List - education.uwmedicine.org...Career Advisors FAQ List For questions regarding career counseling, please contact SOM Career Advising at medadv@uw.edu Updated

16

5 4 3 2 1Highly Important Less Important

●●

●●●

If a student is strongly interested in Family Medicine, please rate how important each of the following activities and performance factors are for a successful residency match.

Achieving 4 or more "Honors" in required clerkship rotations ●

Achieving "Honors" in this specialty's clerkship rotation ●

Doing an away rotation in this specialtyDoing a summer research project in this specialty ●

Doing a preceptorship in this specialty ●Doing a UWSOM Explore & Focus Sub-I in this specialty

3 letters of recommendation from physicians in this specialty ●

Express a strong interest in academic medicine ●

1 letter of reference from someone in this specialty1 letter of recommendation from a department chair in this specialty ●

Having a Step I USMLE score above 235 ●Election to AOA ●

Having a published abstract or poster presentationHaving at least one published paper in a peer reviewed journal ●

GHIP or other international experienceHaving an interest in practicing in a rural or underserved community

Having a Master's Degree (eg.MPH)Being bilingual

Participating in community service

Doing a RUOP or other experience in an underserved communityElection to Gold Humanism Honor Society (GHHS) ●

Page 17: Career Advisors FAQ List - education.uwmedicine.org...Career Advisors FAQ List For questions regarding career counseling, please contact SOM Career Advising at medadv@uw.edu Updated

17

Length of Residency Training

3 years (a small number of 4 year programs exist)

Rate Level of Competitiveness of Specialty

Low to Medium

Mean # of Programs Applied to in this Specialty

30.7

National Characteristics of Entering Family Medicine ResidentsCategorical vs. Advanced Program Median Salaries

# of U.S. M.D. Applicants in this Specialty # of Positions Available

4,949 3,629

For more information, please visit the AAMC's Careers in Medicine: Family Medicine website.

CategoricalClinical Practice starting salary: $241,500Academic Med. Assistant Prof: $201,000Academic Med. Full Prof: $220,000

Mean USMLE Step 1 Score Mean USMLE Step 2 CK Score

220 237

Return to Table of Contents

At what point in the interview season should a student become concerned about their application to Family Medicine residency programs?

If less than 5 interviews by mid-October.

If a student is concerned about their application status, what should they do?

Contact your FM career advisor.

Residency Application Process - Frequently Asked QuestionsHow many programs should a student apply to in order to be successful in matching in Family Medicine?

15 for the average applicant. Try to get around 12 interviews, 10 on rank list.

Recognizing that it can vary from year to year, nationally, when is the typical interview season for Family Medicine?

Most interviews programs interview in November and December. Some start in early in mid-October and some go through January. Plan 4-6 weeks.

Page 18: Career Advisors FAQ List - education.uwmedicine.org...Career Advisors FAQ List For questions regarding career counseling, please contact SOM Career Advising at medadv@uw.edu Updated

18

Career Advisor

Dr. Doug Paauw, Dr. Jenny Wright

Departmental AdvisorsDrs. Daniel Cabrera, Chris Knight, Susan Merel, Shobah Stack, Kathi Sleavin

Internal MedicinePreferred Method of Contact Contact Information

Email Initial inquiries email only [email protected] emails - [email protected], [email protected].

Osler ClubDr. Doug Paauw

[email protected]

Advisors Assigned by Department Departmental Career Advising Website

Yeshttps://imstudents.uw.edu/career-advising/career-advising

Student Interest Group (S.I.G.) S.I.G. Contact Information

Do you recommend that students take an elective in Internal Medicine at another institution?

No. Students should only do an away elective if they have a compelling reason to match at that specific site.

Are there any electives outside this specialty that you recommend to students interested in matching into Internal Medicine?

For students interested in matching into Internal Medicine, our list of recommended electives are: Strongly Recommended: Cardiology, Clinical Respiratory Disease (Pulmonary). Recommended: ClinicalHIV, ENT, Urology, Rheumatology, Orthopaedics - Outpatient/Sports medicine - Not OR

Which UWSOM Sub-Is help students prepare for residency in Internal Medicine?

Any subinternship in IM - EXCEPT the Subinternship in Neurocritical Care Medicine.

Which Advanced Patient Care clerkships help students prepare for residency in Internal Medicine? Dermatology, Ortho/Sports Medicine (for primary care IM), Primary Care (for Primary Care IM), Advanced Inpatient Geriatrics or Advanced Outpatient Geriatrics (Medicine), Palliative Care, Opioid Use Disorder, Outpatient Cardiology. There are many other APC's in IM; any of these are recommended for students matching in IM.

Frequently Asked QuestionsWhich clerkships are required for students to match in Internal Medicine?

Students matching in IM are encouraged to do their APC Subinternship in one of the Medicine APC SubIs.

Page 19: Career Advisors FAQ List - education.uwmedicine.org...Career Advisors FAQ List For questions regarding career counseling, please contact SOM Career Advising at medadv@uw.edu Updated

19

Do you have any additional recommendations for students who are committed to matching in this specialty?

Do well on your third year clerkships. We are a very straightforward match, so students shouldn't worry about needing away electives or research.

Please list the most common mistakes students make when they apply to your specialty:

We are a very straightforward match. Students across a broad spectrum of competitiveness match into IM residency programs. Students need to think geographically broadly when they apply, there are only 2 IM residency programs in Seattle, so not everyone will match into a Seattle program.

Page 20: Career Advisors FAQ List - education.uwmedicine.org...Career Advisors FAQ List For questions regarding career counseling, please contact SOM Career Advising at medadv@uw.edu Updated

20

5 4 3 2 1Highly Important Less Important

Doing a preceptorship in this specialty ●Doing a UWSOM Explore & Focus Sub-I in this specialty

If a student is strongly interested in Internal Medicine, please rate how important each of the following activities and performance factors are for a successful residency match.

Achieving 4 or more "Honors" in required clerkship rotations ●

Achieving "Honors" in this specialty's clerkship rotation

1 letter of reference from someone in this specialty1 letter of recommendation from a department chair in this specialty

Doing an away rotation in this specialty ●Doing a summer research project in this specialty ●

Having a published abstract or poster presentation ●Having at least one published paper in a peer reviewed journal

3 letters of recommendation from physicians in this specialty

Express a strong interest in academic medicine ●

Having a Master's Degree (eg.MPH) ●Being bilingual ●

Having a Step I USMLE score above 235 ●Election to AOA ●

Doing a RUOP or other experience in an underserved community ●Election to Gold Humanism Honor Society (GHHS) ●

GHIP or other international experience ●Having an interest in practicing in a rural or underserved community

Participating in community service ●

Page 21: Career Advisors FAQ List - education.uwmedicine.org...Career Advisors FAQ List For questions regarding career counseling, please contact SOM Career Advising at medadv@uw.edu Updated

21

Length of Residency Training

3 years

Rate Level of Competitiveness of Specialty

Low to Medium

Mean # of Programs Applied to in this Specialty

32.1

CategoricalClinical Practice starting salary: $261,000Academic Med. Assistant Prof: $211,000Academic Med. Full Prof: $244,000

Mean USMLE Step 1 Score Mean USMLE Step 2 CK Score

233 (This is Mean score, IM matches a broad range of Step 1 scores)

246 (This is Mean score, IM matches a broad range of Step 2 scores)

National Characteristics of Entering Internal Medicine ResidentsCategorical vs. Advanced Program Median Salaries

Residency Application Process - Frequently Asked QuestionsHow many programs should a student apply to in order to be successful in matching in Internal Medicine?Depends on how competetive the student is: 20-60 IM helps students select number of programs to interview.

Recognizing that it can vary from year to year, nationally, when is the typical interview season for Internal Medicine?October-first half of January. Peak time is Nov/Dec. Many programs break for the last week of December between holidays.

# of U.S. M.D. Applicants in this Specialty # of Positions Available

11,171 7,542

For more information, please visit the AAMC's Careers in Medicine: Internal Medicine website.

Additional information:

Students interested in IM should contact Kathi Sleavin, [email protected] to be added to our list of interested students for the upcoming Match year.

Return to Table of Contents

At what point in the interview season should a student become concerned about their application to Internal Medicine residency programs?Mid-November, if students haven't yet received 10 invitations for IM and 8 for prelim medicine.

At what point in the interview season should a student become concerned about their application to Internal Medicine residency programs? If students have less than 10 interviews by Mid-November, we should be contacted. But less than 10 can be fine if the programs student is interviewing at are matchable programs for the student. And, some highly competetive applicants can get by with less interviews, and we tell them this in advance.

Page 22: Career Advisors FAQ List - education.uwmedicine.org...Career Advisors FAQ List For questions regarding career counseling, please contact SOM Career Advising at medadv@uw.edu Updated

22

Career Advisor

Dr. Richard Ellenbogen

Departmental AdvisorsJeff Ojemann MD, Louis J. Kim MD, Manuel Ferreira MD, PhD

Neurological SurgeryPreferred Method of Contact Contact Information

Phone 206.744.9321

Do you recommend that students take an elective in Neurological Surgery at another institution?Yes

Does not have its own group--try PNIG or Surgery Interest Group N/A

Advisors Assigned by Department Departmental Career Advising Website

N/A http://neurosurgery.washington.edu/education/residency/

Student Interest Group (S.I.G.) S.I.G. Contact Information

Are there any electives outside this specialty that you recommend to students interested in matching into Neurological Surgery? Most clerkships are pertinent

Frequently Asked QuestionsWhich clerkships are required for students to match in Neurological Surgery?

Neurosurgery Clerkship.

Which UWSOM Sub-Is help students prepare for residency in Neurological Surgery?

Neurocritical Care Medicine (Neurosurgical ICU), Surgical Intensive Care Unit (Surgical ICU).

Which Advanced Patient Care clerkships help students prepare for residency in Neurological Surgery?

Pain Medicine Clerkship, Palliative Care, Medical Consultation.

Do you have any additional recommendations for students who are committed to matching in this specialty?

Discuss with neurological surgeons early in your 3rd year if possible.

Please list the most common mistakes students make when they apply to your specialty:

Not getting advice on sub-I selection and specialty selection.

Page 23: Career Advisors FAQ List - education.uwmedicine.org...Career Advisors FAQ List For questions regarding career counseling, please contact SOM Career Advising at medadv@uw.edu Updated

23

5 4 3 2 1Highly Important Less Important

●●●●

●●

●●

If a student is strongly interested in Neurological Surgery, please rate how important each of the following activities and performance factors are for a successful residency match.

Achieving 4 or more "Honors" in required clerkship rotations

Achieving "Honors" in this specialty's clerkship rotation

Doing an away rotation in this specialtyDoing a summer research project in this specialty

Doing a preceptorship in this specialtyDoing a UWSOM Explore & Focus Sub-I in this specialty.

3 letters of recommendation from physicians in this specialty

Express a strong interest in academic medicine

1 letter of reference from someone in this specialty1 letter of recommendation from a department chair in this specialty

Having a Step I USMLE score above 235Election to AOA

Having a published abstract or poster presentationHaving at least one published paper in a peer reviewed journal

GHIP or other international experience ●Having an interest in practicing in a rural or underserved community ●

Having a Master's Degree (eg.MPH) ●Being bilingual ●

Participating in community service ●

Doing a RUOP or other experience in an underserved community ●Election to Gold Humanism Honor Society (GHHS)

Page 24: Career Advisors FAQ List - education.uwmedicine.org...Career Advisors FAQ List For questions regarding career counseling, please contact SOM Career Advising at medadv@uw.edu Updated

24

Length of Residency Training

7 years

Rate Level of Competitiveness of Specialty

High

Mean # of Programs Applied to in this Specialty

68.1

National Characteristics of Entering Neurological Surgery ResidentsCategorical vs. Advanced Program Median Salaries

# of U.S. M.D. Applicants in this Specialty # of Positions Available

305 225

For more information, please visit the AAMC's Careers in Medicine: Neurological Surgery website.

CategoricalAcademic Med. Assistant Prof: $537,000Academic Med. Full Prof: $685,000

Mean USMLE Step 1 Score Mean USMLE Step 2 CK Score

245 249

Return to Table of Contents

At what point in the interview season should a student become concerned about their application to Neurological Surgery residency programs?

The first week of December.

If a student is concerned about their application status, what should they do?

Contact residency coordinator

Residency Application Process - Frequently Asked QuestionsHow many programs should a student apply to in order to be successful in matching in Neurological Surgery?

20

Recognizing that it can vary from year to year, nationally, when is the typical interview season for Neurological Surgery?

October to January

Page 25: Career Advisors FAQ List - education.uwmedicine.org...Career Advisors FAQ List For questions regarding career counseling, please contact SOM Career Advising at medadv@uw.edu Updated

25

Career Advisor

Dr. Eric Kraus

Departmental Advisors

Dr. James Owens (Child Neurology)

Are there any electives outside this specialty that you recommend to students interested in matching into Neurology?

Ophthalmology, Neurosurgery, Otolaryngology, Rehabilitation Medicine, Radiology

Frequently Asked QuestionsWhich clerkships are required for students to match in Neurology?

Neurology

Which UWSOM Sub-Is help students prepare for residency in Neurology?

Subinternship in Neurocritical Care Medicine (Neurosurgical ICU, conjoint Med/Anesthesia), Ward or Hospital Medicine.

Which Advanced Patient Care clerkships help students prepare for residency in Neurology?

Consultation Liaison Psychiatry, Advanced Inpatient Geriatrics or Advanced Outpatient Geriatrics (Medicine), Palliative Care, Medical Consultation.

NeurologyPhone Number Contact Information

206.598.0216 [email protected]

Do you recommend that students take an elective in Neurology at another institution?

Yes, if a student is interested in matching at a specific institution, and would like to audition at that institution.Not required for speciality.

Psychiatry/Neurology Interest Group (PNIG) Dr. Eric Kraus - [email protected]

Advisors Assigned by Department Departmental Career Advising Website

No N/A

Student Interest Group (S.I.G.) S.I.G. Contact Information

Page 26: Career Advisors FAQ List - education.uwmedicine.org...Career Advisors FAQ List For questions regarding career counseling, please contact SOM Career Advising at medadv@uw.edu Updated

26

Do you have any additional recommendations for students who are committed to matching in this specialty?

Do more than one Neurology rotation at home or other institution. UW offers EEG and pediatric neurology electives. Meet with a departmental advisor. Pass both USMLE exams on the first try. Have a well-developed idea about the practice of Neurology.

Please list the most common mistakes students make when they apply to your specialty:

Applying late in the process; it's best to submit applications early in September. Not having a letter from a neurologist. A personal statement that is too casual, flush with bragging, has typos, or is poorly worded (ask more than one person to proofread, ideally someone who has reviewed a number of personal statements).

Page 27: Career Advisors FAQ List - education.uwmedicine.org...Career Advisors FAQ List For questions regarding career counseling, please contact SOM Career Advising at medadv@uw.edu Updated

27

5 4 3 2 1Highly Important Less Important

●●

●●

Participating in community service ●

Doing a RUOP or other experience in an underserved communityElection to Gold Humanism Honor Society (GHHS) ●

GHIP or other international experienceHaving an interest in practicing in a rural or underserved community

Having a Master's Degree (eg.MPH) ●Being bilingual ●

Having a Step I USMLE score above 235Election to AOA

Having a published abstract or poster presentation ●Having at least one published paper in a peer reviewed journal ●

3 letters of recommendation from physicians in this specialty ●

Express a strong interest in academic medicine ●

1 letter of reference from someone in this specialty1 letter of recommendation from a department chair in this specialty ●

Doing an away rotation in this specialty ●Doing a summer research project in this specialty ●

Doing a preceptorship in this specialtyDoing a UWSOM Explore & Focus Sub-I in this specialty.

Achieving "Honors" in this specialty's clerkship rotation

If a student is strongly interested in Neurology, please rate how important each of the following activities and performance factors are for a successful residency match.

Achieving 4 or more "Honors" in required clerkship rotations

Page 28: Career Advisors FAQ List - education.uwmedicine.org...Career Advisors FAQ List For questions regarding career counseling, please contact SOM Career Advising at medadv@uw.edu Updated

28

Length of Residency Training

4 years

Rate Level of Competitiveness of Specialty

Medium

Mean # of Programs Applied to in this Specialty

26.1

Return to Table of Contents

At what point in the interview season should a student become concerned about their application to Neurology residency programs?

If they have received fewer than 5 interviews by Mid-October

If a student is concerned about their application status, what should they do?

First, contact the residency program. Second, contact the career advisor

Residency Application Process - Frequently Asked QuestionsHow many programs should a student apply to in order to be successful in matching in Neurology?

10-15 interviews. Depending on your competitiveness this may mean many more programs in ERAS.

Recognizing that it can vary from year to year, nationally, when is the typical interview season for Neurology?

Late October to January

# of U.S. M.D. Applicants in this Specialty # of Positions Available

973 552

For more information, please visit the AAMC's Careers in Medicine: Neurology website.

Both availableClinical Practice starting salary: $301,000Academic Med. Assistant Prof: $211,000Academic Med. Full Prof: $251,000

Mean USMLE Step 1 Score Mean USMLE Step 2 CK Score

231 242

National Characteristics of Entering Neurology ResidentsCategorical vs. Advanced Program Median Salaries

Page 29: Career Advisors FAQ List - education.uwmedicine.org...Career Advisors FAQ List For questions regarding career counseling, please contact SOM Career Advising at medadv@uw.edu Updated

29

Career Advisor

Dr. Vicki Mendiratta

Departmental Advisors

N/A

Are there any electives outside this specialty that you recommend to students interested in matching into Obstetrics-Gynecology?

Consider: Female Urology, ID, Cardiology/Clinical Respiratory Disease (Pulmonary)/Critical Care, basic Anesthesia clerkship, Radiology.

Frequently Asked QuestionsWhich clerkships are required for students to match in Obstetrics-Gynecology?

Core 3rd year clerkships. A 4th year Sub-I in OB/GYN is not required but either a UW or a Visiting away Sub-I is highly encouraged. Dr. Mendiratta will help students to determine what might be best for them.

Which UWSOM Sub-Is help students prepare for residency in Obstetrics-Gynecology? OB/GYN: all 3 of our sub-Is: GYN ONC, High risk OB, GYN subspecialtiesOutside of OB/GYN: Anesthesia Critical Care APC in Medicine, or Subinternship in Critical Care Medicine taken as APC Medical Consultation.

Which Advanced Patient Care clerkships help students prepare for residency in Obstetrics-Gynecology? Recommend taking an OB/GYN sub-I. Outside OB/GYN: Critical Care, Medical Consultation.

Obstetrics-GynecologyPreferred Method of Contact Contact Information

Email [email protected]

Do you recommend that students take an elective in Obstetrics-Gynecology at another institution?

All student strongly encouraged to consider away OB/GYN sub-I but a definate for "at risk" students

OB/GYN Interest GroupDr. Vicki Mendiratta

[email protected]

Advisors Assigned by Department Departmental Career Advising Website

Nohttp://depts.washington.edu/obgyn/education/third-year-clerkship/4th-year-info.html

Student Interest Group (S.I.G.) S.I.G. Contact Information

Page 30: Career Advisors FAQ List - education.uwmedicine.org...Career Advisors FAQ List For questions regarding career counseling, please contact SOM Career Advising at medadv@uw.edu Updated

30

Do you have any additional recommendations for students who are committed to matching in this specialty?

1) Students applying for OB/GYN must have a 1st time PASS for both Step 1 and 2.2) Ask for letters of recommendation as you go from a variety of people - in OB, we like diversity among the letters - not just from OB/GYNs . You will need 3-4 letters total, sometimes "Chair" letter is required as well3) Strive to do well in all required clerkships - # of Honors in the 3rd year is an important criteria for competitive residency programs and particularly in the West region.4.) Meet with me early (1st/2nd yr), then frequently during 3rd and early 4th.

Please list the most common mistakes students make when they apply to your specialty:Look all around the country - not just in the West. There are limited programs in the West relative to the rest of the nation. Some students will benefit from Dual applying depending on their unique circumstance. The "Scramble/SOAP" really does not yield open OB spots, save some Preliminary spots. Dr. Mendiratta will advise students who should strongly consider dual applying. Applying for programs that are out of their range for their competitiveness.

Page 31: Career Advisors FAQ List - education.uwmedicine.org...Career Advisors FAQ List For questions regarding career counseling, please contact SOM Career Advising at medadv@uw.edu Updated

31

5 4 3 2 1Highly Important Less Important

●●

●●

●Participating in community service

Doing a RUOP or other experience in an underserved communityElection to Gold Humanism Honor Society (GHHS) ●

GHIP or other international experience ●Having an interest in practicing in a rural or underserved community

Having a Master's Degree (eg.MPH)Being bilingual

Having a Step I USMLE score above 235 ●Election to AOA ●

Having a published abstract or poster presentationHaving at least one published paper in a peer reviewed journal

3 letters of recommendation from physicians in this specialty

Express a strong interest in academic medicine

1 letter of reference from someone in this specialty1 letter of recommendation from a department chair in this specialty ●

Doing an away rotation in this specialty ●Doing a summer research project in this specialty

Doing a preceptorship in this specialty ●Doing a UWSOM Explore & Focus Sub-I in this specialty

Achieving "Honors" in this specialty's clerkship rotation

If a student is strongly interested in Obstetrics-Gynecology, please rate how important each of the following activities and performance factors are for a successful residency match.

Achieving 4 or more "Honors" in required clerkship rotations ●

Page 32: Career Advisors FAQ List - education.uwmedicine.org...Career Advisors FAQ List For questions regarding career counseling, please contact SOM Career Advising at medadv@uw.edu Updated

32

Length of Residency Training

4 years

Rate Level of Competitiveness of Specialty

Medium

Mean # of Programs Applied to in this Specialty

51.3

Return to Table of Contents

At what point in the interview season should a student become concerned about their application to Obstetrics-Gynecology residency programs?

Mid October if < 10 interviews confirmed

If a student is concerned about their application status, what should they do?

Contact Dr. Mendiratta.

Residency Application Process - Frequently Asked QuestionsHow many programs should a student apply to in order to be successful in matching in Obstetrics-Gynecology?

Variable depending on competitiveness of student and types of programs applying to. At least 50

Recognizing that it can vary from year to year, nationally, when is the typical interview season for Obstetrics-Gynecology?

November and December; There are some in October and January, but majority in Nov/Dec.

# of U.S. M.D. Applicants in this Specialty # of Positions Available

1,894 1,336

For more information, please visit the AAMC's Careers in Medicine: Obstetrics and Gynecology website.

Categorical Clinical Practice starting salary: $324,000Academic Med. Assistant Prof: $258,000Academic Med. Full Prof: $304,000

Mean USMLE Step 1 Score Mean USMLE Step 2 CK Score

230 247

National Characteristics of Entering Obstetrics-Gynecology ResidentsCategorical vs. Advanced Program Median Salaries

Page 33: Career Advisors FAQ List - education.uwmedicine.org...Career Advisors FAQ List For questions regarding career counseling, please contact SOM Career Advising at medadv@uw.edu Updated

33

Career Advisor

Dr. Courtney Francis

Departmental Advisors

Dr. Michelle Cabrera

Are there any electives outside this specialty that you recommend to students interested in matching into Ophthalmology?

Emergency Medicine, Radiology, Otolaryngology, Endocrinology, Dermatology and Rheumatology are sometimes helpful to Ophthalmology residents, but are not necessary.

Frequently Asked QuestionsWhich clerkships are required for students to match in Ophthalmology? Core clerkships and at least one ophthalmology rotation. Ophthalmology 681 (Harborview/UW Eye Institute), Ophthalmology 685 (VA Puget Sound), Ophthalmology 683 (Seattle Children's Hospital) and Ophthalmology 695 (Advanced Ophthalmology) are recommended.

Which UWSOM Sub-Is help students prepare for residency in Ophthalmology?

Ward or Hospital Medicine, General Surgery.

Which Advanced Patient Care clerkships help students prepare for residency in Ophthalmology?

Any of personal interest.

OphthalmologyPreferred Method of Contact Contact Information

Email [email protected]

Do you recommend that students take an elective in Ophthalmology at another institution?

An away elective at another institution may sometimes be helpful for students interested in a particular program; however, away electives are not necessary for matching.

Ophthalmology Interest Group Dr. Courtney Francis - [email protected]

Advisors Assigned by Department Departmental Career Advising Website

No N/A

Student Interest Group (S.I.G.) S.I.G. Contact Information

Page 34: Career Advisors FAQ List - education.uwmedicine.org...Career Advisors FAQ List For questions regarding career counseling, please contact SOM Career Advising at medadv@uw.edu Updated

34

Do you have any additional recommendations for students who are committed to matching in this specialty?

Meet with a departmental advisor as early as you think you may have an interest in Ophthalmology given the January match date. Core clerkship grades and USMLE scores are the most important components to matching successfully. This past year, the mean USMLE Step 1 score of a matched applicant was 244. Participation in a research project is also important.

Please list the most common mistakes students make when they apply to your specialty:

Academic performance, particularly on core clinical clerkships and the USMLE Step 1, may limit a student's ability to match successfully in Ophthalmology. Meeting with a career advisor too late and limited time in Seattle may also make matching more challenging.

Page 35: Career Advisors FAQ List - education.uwmedicine.org...Career Advisors FAQ List For questions regarding career counseling, please contact SOM Career Advising at medadv@uw.edu Updated

35

5 4 3 2 1Highly Important Less Important

●●

●●

Participating in community service ●

Doing a RUOP or other experience in an underserved community ●Election to Gold Humanism Honor Society (GHHS) ●

GHIP or other international experience ●Having an interest in practicing in a rural or underserved community

Having a Master's Degree (eg.MPH)Being bilingual ●

Having a Step I USMLE score above 235Election to AOA

Having a published abstract or poster presentationHaving at least one published paper in a peer reviewed journal ●

3 letters of recommendation from physicians in this specialty

Express a strong interest in academic medicine ●

1 letter of reference from someone in this specialty1 letter of recommendation from a department chair in this specialty

Doing an away rotation in this specialtyDoing a summer research project in this specialty

Doing a preceptorship in this specialtyDoing a UWSOM Explore & Focus Sub-I in this specialty.

Achieving "Honors" in this specialty's clerkship rotation

If a student is strongly interested in Ophthalmology, please rate how important each of the following activities and performance factors are for a successful residency match.

Achieving 4 or more "Honors" in required clerkship rotations

Page 36: Career Advisors FAQ List - education.uwmedicine.org...Career Advisors FAQ List For questions regarding career counseling, please contact SOM Career Advising at medadv@uw.edu Updated

36

Length of Residency Training

4 years (including the PGY-1 year)

Rate Level of Competitiveness of Specialty

High

Mean # of Programs Applied to in this Specialty

75.0

Return to Table of Contents

At what point in the interview season should a student become concerned about their application to Ophthalmology residency programs?

Fewer than 3-4 interview offers by mid November. 8-10 interviews maximizes the likelihood of matching.

If a student is concerned about their application status, what should they do?

Meet with the department advisor prior to submitting the application.

Residency Application Process - Frequently Asked QuestionsHow many programs should a student apply to in order to be successful in matching in Ophthalmology?40-60

Recognizing that it can vary from year to year, nationally, when is the typical interview season for Ophthalmology?

Mid-late October through December. Internship interviews often go into early February.

Total # of applicants participating # of Positions Offered

740 485

For more information, please visit the American Academy of Ophthalmology website.

Advanced/Categorical (transitioning to integrated internships)

Academic Med. Assistant Prof: $264,000Academic Med. Full Prof: $349,000

For more information, please visit the AAMC's Careers in Medicine: Ophthalmology website.Mean USMLE Step 1 Score Mean USMLE Step 2 CK Score

244 Info not available

National Characteristics of Entering Ophthalmology ResidentsCategorical vs. Advanced Program Median Salaries

Page 37: Career Advisors FAQ List - education.uwmedicine.org...Career Advisors FAQ List For questions regarding career counseling, please contact SOM Career Advising at medadv@uw.edu Updated

40

Career Advisor

Dr. Lisa Taitsman

Departmental AdvisorsDrs. Jennifer Bauer, Todd Blumberg, Michael Githens, Mia Hagen, Nick Iannuzzi, Chris Kweon, Jared Harwood and other orthopaedic faculty

Frequently Asked QuestionsWhich clerkships are required for students to match in Orthopaedics?

Core Clerkships.Orthopaedics- not officially required, but strongly recommended

Which UWSOM Sub-Is help students prepare for residency in Orthopaedics?

General Surgery or other surgical subspecialty

Which Advanced Patient Care clerkships help students prepare for residency in Orthopaedics? Sports Medicine, Advanced Inpatient Geriatrics or Advanced Outpatient Geriatrics (Medicine), Medical Consultation.Other APC's or Subinternships in Internal Medicine or Family Medicine or Surgery.

OrthopaedicsPreferred Method of Contact Contact Information

via coordinator Plinette Sanchez [email protected]

Do you recommend that students take an elective in Orthopaedics at another institution?

Yes, most students average 2 away rotations.

Orthopaedic Surgery Interest Group and Sports Medicine Interest Group (OSSMIG)

Dr. Mia Hagenhttp://www.orthop.washington.edu/?q=ossmig/orthopaedic-surgery-and-sports-medicine-interest-group-ossmig.html

Advisors Assigned by Department Departmental Career Advising Website

Yes http://www.orthop.washington.edu/?q=ossmig/orthopaedic-surgery-and-sports-medicine-interest-group-ossmig.html

Student Interest Group (S.I.G.) S.I.G. Contact Information

Page 38: Career Advisors FAQ List - education.uwmedicine.org...Career Advisors FAQ List For questions regarding career counseling, please contact SOM Career Advising at medadv@uw.edu Updated

41

Do you have any additional recommendations for students who are committed to matching in this specialty?

Should have above average USMLE scores. Honors in core clerkships is very important. Research interest and experience.Outside interests/ activities. Apptitude for procedures/ technical skills.

Please list the most common mistakes students make when they apply to your specialty:

Having realistic expectations regarding matching as orthopaedics is very competitive. It is important to select programs for away rotations that you would want to train in but also where you have a reasonable chance of matching.EVERY medical student applying in orthopaedics needs a backup plan in the event of not matching.

Are there any electives outside this specialty that you recommend to students interested in matching into Orthopaedics?Nothing is formally required. Other surgical rotation can be useful, especially in decision making regarding your career.Other non-surgical clerkships such as radiology, neurology, PM&R, sports medicine and rheumatology can also be helpful from an educational standpoint for orthopaedics. Internal medicine, family medicine, geriatrics, and general surgery are all very valuable for learning how to care for patients.

Page 39: Career Advisors FAQ List - education.uwmedicine.org...Career Advisors FAQ List For questions regarding career counseling, please contact SOM Career Advising at medadv@uw.edu Updated

42

5 4 3 2 1Highly Important Less Important

●●●

●●●

●●●

● Participating in community service ●

Doing a RUOP or other experience in an underserved communityElection to Gold Humanism Honor Society (GHHS)

GHIP or other international experienceHaving an interest in practicing in a rural or underserved community

Having a Master's Degree (eg.MPH)Being bilingual

Having a Step I USMLE score above 235Election to AOA

Having a published abstract or poster presentationHaving at least one published paper in a peer reviewed journal

3 letters of recommendation from physicians in this specialty

Express a strong interest in academic medicine ●

1 letter of reference from someone in this specialty1 letter of recommendation from a department chair in this specialty ●

Doing an away rotation in this specialtyDoing a summer research project in this specialty ●

Doing a preceptorship in this specialty

Doing a UWSOM Explore & Focus Sub-I in this specialty

Achieving "Honors" in this specialty's clerkship rotation

If a student is strongly interested in Orthopaedics, please rate how important each of the following activities and performance factors are for a successful residency match.

Achieving 4 or more "Honors" in required clerkship rotations

Page 40: Career Advisors FAQ List - education.uwmedicine.org...Career Advisors FAQ List For questions regarding career counseling, please contact SOM Career Advising at medadv@uw.edu Updated

43

Length of Residency Training

5 years

Rate Level of Competitiveness of Specialty

High

Mean # of Programs Applied to in this Specialty

74.8

Return to Table of Contents

At what point in the interview season should a student become concerned about their application to Orthopaedics residency programs?

Again, no data. Focus on doing an outstanding job on rotations- at UW and when away.Best to anticipate up front and apply to a wide range of programs with a back up plan in place from the beginning.There is no set time that programs notify about applications and no minimum number of interviews that guarantee anything.

If a student is concerned about their application status, what should they do?

Discuss with their advisor. The earlier the better.

Residency Application Process - Frequently Asked QuestionsHow many programs should a student apply to in order to be successful in matching in Orthopaedics?

There is no data about this. I have always heard 40-50. It is important to apply to a range of programs to improve chances of matching.

Recognizing that it can vary from year to year, nationally, when is the typical interview season for Orthopaedics?

The busiest interview months are December and January.

# of U.S. M.D. Applicants in this Specialty # of Positions Available

1,172 742

For more information, please visit: AAMC's Careers in Medicine: Orthopaedic Surgery website.

Categorical Clinical Practice starting salary: $533,000Academic Med. Assistant Prof: $458,000Academic Med. Full Prof: $586,000

Mean USMLE Step 1 Score Mean USMLE Step 2 CK Score

248 255

National Characteristics of Entering Orthopaedics ResidentsCategorical vs. Advanced Program Median Salaries

Page 41: Career Advisors FAQ List - education.uwmedicine.org...Career Advisors FAQ List For questions regarding career counseling, please contact SOM Career Advising at medadv@uw.edu Updated

44

Career AdvisorDr. Ian Humphreys

Departmental Advisors

Drs. Maya Sardesai, Tanya Meyer, Jake Dahl, Sanjay Parikh, Mark Whipple, Cliff Hume

Do you have any additional recommendations for students who are committed to matching in this specialty?Be able to talk about your research in depth. Likely the student will not be the first author on a peer reviewed journal, but in all cases the student should be well versed in the research project and be able to discuss the outcomes of the research.

Please list the most common mistakes students make when they apply to your specialty:Recognize early that Oto/HNS is a competitive residency. Early planning, and contact with faculty in our department is very helpful. Also, not knowing your research.

Are there any electives outside this specialty that you recommend to students interested in matching into Otolaryngology? Neurosurgery, Radiology, General Surgery, Thoracic Surgery, GI Medicine, Clinical Respiratory Disease (Pulmonary), Surgical ICU (recommended for education prior to residency in Oto/HNS).

Frequently Asked QuestionsWhich clerkships are required for students to match in Otolaryngology? Oto/HNS 4 week clerkship

Which UWSOM Sub-Is help students prepare for residency in Otolaryngology? Surgical Intensive Care Unit (Surgical ICU), Thoracic Surgery.

Which Advanced Patient Care clerkships help students prepare for residency in Otolaryngology? Otolaryngology APC.

OtolaryngologyPreferred Method of Contact Contact InformationEmail [email protected]

Do you recommend that students take an elective in Otolaryngology at another institution?If a student is very interested in a particular program, then it is reasonable to take an away rotation at that institution, and it will function as a "4-week interview". This is helpful, especially if the student might not get an interview offer otherwise.

Otolaryngology Head and Neck Surgery Interest Group (OHNSIG)Dr. Ian [email protected]

Advisors Assigned by Department Departmental Career Advising Website

No N/A

Student Interest Group (S.I.G.) S.I.G. Contact Information

Page 42: Career Advisors FAQ List - education.uwmedicine.org...Career Advisors FAQ List For questions regarding career counseling, please contact SOM Career Advising at medadv@uw.edu Updated

45

5 4 3 2 1Highly Important Less Important

●●

●●

●Participating in community service

Doing a RUOP or other experience in an underserved communityElection to Gold Humanism Honor Society (GHHS) ●

GHIP or other international experience ●Having an interest in practicing in a rural or underserved community

Having a Master's Degree (eg.MPH) ●Being bilingual

Having a Step I USMLE score above 235Election to AOA ●

Having a published abstract or poster presentationHaving at least one published paper in a peer reviewed journal ●

3 letters of recommendation from physicians in this specialty

Express a strong interest in academic medicine

1 letter of reference from someone in this specialty1 letter of recommendation from a department chair in this specialty

Doing an away rotation in this specialtyDoing a summer research project in this specialty ●

Doing a preceptorship in this specialty ●Doing a UWSOM Explore & Focus Sub-I in this specialty

Achieving "Honors" in this specialty's clerkship rotation

If a student is strongly interested in Otolaryngology, please rate how important each of the following activities and performance factors are for a successful residency match.

Achieving 4 or more "Honors" in required clerkship rotations

Page 43: Career Advisors FAQ List - education.uwmedicine.org...Career Advisors FAQ List For questions regarding career counseling, please contact SOM Career Advising at medadv@uw.edu Updated

46

Length of Residency Training

5 years

Rate Level of Competitiveness of Specialty

High

Mean # of Programs Applied to in this Specialty

39.2

Return to Table of Contents

At what point in the interview season should a student become concerned about their application to Otolaryngology residency programs?

If there are not at least 5 interview invitations by late October.

If a student is concerned about their application status, what should they do?

Contact Ian Humphreys or their Oto/HNS advisor.

Residency Application Process - Frequently Asked QuestionsHow many programs should a student apply to in order to be successful in matching in Otolaryngology?

40-50

Recognizing that it can vary from year to year, nationally, when is the typical interview season for Otolaryngology?

November-January

# of U.S. M.D. Applicants in this Specialty # of Positions Available

570 315

For more information, please visit the AAMC's Careers in Medicine: Otolaryngology website.

Categorical Clinical Practice starting salary: $405,000Academic Med. Assistant Prof: $330,000Academic Med. Full Prof: $438,000

Mean USMLE Step 1 Score Mean USMLE Step 2 CK Score

248 254

National Characteristics of Entering Otolaryngology ResidentsCategorical vs. Advanced Program Median Salaries

Page 44: Career Advisors FAQ List - education.uwmedicine.org...Career Advisors FAQ List For questions regarding career counseling, please contact SOM Career Advising at medadv@uw.edu Updated

47

Career Advisor

Dr. Mara Rendi, Dr. Jose Mantilla

Departmental Advisors

N/A

Do you have any additional recommendations for students who are committed to matching in this specialty?Just do well on the path or lab medicine elective.

Please list the most common mistakes students make when they apply to your specialty: Not exploring all the options in pathology- i.e. Whether to apply Anatomic Pathology (AP)/Clinical Pathology (CP), AP only, CP only, or AP/Neuropathology and AP/Hematopathology. Talk to your path advisor/mentors about your interests and how best to apply.

Are there any electives outside this specialty that you recommend to students interested in matching into Pathology?

None specifically but taking more clinical electives will help you in pathology so you understand the clinician's needs.

Frequently Asked QuestionsWhich clerkships are required for students to match in Pathology? An elective in anatomic pathology and/or lab medicine- should be at least 4 weeks total spent in AP/Lab Medicine.

Which UWSOM Sub-Is help students prepare for residency in Pathology?

Ward or Hospital Medicine, General Surgery.

Which Advanced Patient Care clerkships help students prepare for residency in Pathology? No specific recommendations - choose based on your interests.

PathologyPreferred Method of Contact Contact Information

Email [email protected]; [email protected]

Do you recommend that students take an elective in Pathology at another institution?

Depends on the student. If you have one place you know you want to be, doing an away rotation is fine. But certainly not necessary.

Pathology Interest GroupDr. Libby Parker

[email protected]

Advisors Assigned by Department Departmental Career Advising Website

No N/A

Student Interest Group (S.I.G.) S.I.G. Contact Information

Page 45: Career Advisors FAQ List - education.uwmedicine.org...Career Advisors FAQ List For questions regarding career counseling, please contact SOM Career Advising at medadv@uw.edu Updated

48

5 4 3 2 1Highly Important Less Important

●●

●●

●Participating in community service

Doing a RUOP or other experience in an underserved community ●Election to Gold Humanism Honor Society (GHHS) ●

GHIP or other international experience ●Having an interest in practicing in a rural or underserved community ●

Having a Master's Degree (eg.MPH) ●Being bilingual ●

Having a Step I USMLE score above 235 ●Election to AOA ●

Having a published abstract or poster presentationHaving at least one published paper in a peer reviewed journal

3 letters of recommendation from physicians in this specialty

Express a strong interest in academic medicine

1 letter of reference from someone in this specialty1 letter of recommendation from a department chair in this specialty

Doing an away rotation in this specialtyDoing a summer research project in this specialty

Doing a preceptorship in this specialty ●Doing a UWSOM Explore & Focus Sub-I in this specialty ●

Achieving "Honors" in this specialty's clerkship rotation

If a student is strongly interested in Pathology, please rate how important each of the following activities and performance factors are for a successful residency match.

Achieving 4 or more "Honors" in required clerkship rotations ●

Page 46: Career Advisors FAQ List - education.uwmedicine.org...Career Advisors FAQ List For questions regarding career counseling, please contact SOM Career Advising at medadv@uw.edu Updated

49

Length of Residency Training

4 years for anatomic & clinical pathology programs; 3 years for anatomic pathology-only programs.

Rate Level of Competitiveness of Specialty

Low. But top programs are competitive and can't get into those unless you are very competitive, but you can get into the others.

Mean # of Programs Applied to in this Specialty

23.5

Return to Table of Contents

At what point in the interview season should a student become concerned about their application to Pathology residency programs?

If by late October/early Nov, student does not have 5 interviews, talk to someone.

If a student is concerned about their application status, what should they do?

Contact Dr. Rendi or anyone in pathology.

Residency Application Process - Frequently Asked QuestionsHow many programs should a student apply to in order to be successful in matching in Pathology?

Around 10

Recognizing that it can vary from year to year, nationally, when is the typical interview season for Pathology?

October - December. October - November is the heaviest time.

# of U.S. M.D. Applicants in this Specialty # of Positions Available

573 601

For more information, please visit the AAMC's Careers in Medicine: Pathology website.

CategoricalAcademic Med. Assistant Prof: $217,000Academic Med. Full Prof: $291,000

Mean USMLE Step 1 Score Mean USMLE Step 2 CK Score

233 242

National Characteristics of Entering Pathology ResidentsCategorical vs. Advanced Program Median Salaries

Page 47: Career Advisors FAQ List - education.uwmedicine.org...Career Advisors FAQ List For questions regarding career counseling, please contact SOM Career Advising at medadv@uw.edu Updated

50

Career Advisor

Dr. Michelle Terry

Departmental Advisors

Drs. Jimmy Beck, Rebekah Burns, Mollie Grow, Susan Hunt, Abena Knight, Caitlin McGrath, Emily Myers, Jordan Symons, Glen Tamura, Michelle Terry

Do you have any additional recommendations for students who are committed to matching in this specialty?Focus on your core rotations and contact us with questions.

Please list the most common mistakes students make when they apply to your specialty:

Failure to apply to a wide variety of programs, sufficient number of programs, don't let their career advisor knowif you are dual applying.

Are there any electives outside this specialty that you recommend to students interested in matching into Pediatrics? Genetics, Pediatric Psychiatry, Pediatric Neurology

Frequently Asked QuestionsWhich clerkships are required for students to match in Pediatrics? None required. Strongly recommended - a 4 week rotation at Seattle Children’s Hospital.

Which UWSOM Sub-Is help students prepare for residency in Pediatrics? Any subinternship within Pediatrics. In addition, an APC within Pediatrics may be done in lieu of a Peds SubI.

Which Advanced Patient Care clerkships help students prepare for residency in Pediatrics? Many options in Pediatrics, including NICU; PICU; Heme/Onc. Also consider Child Psychiatry and Pediatric Surgery.

PediatricsPhone Number Contact Information

[email protected]

Do you recommend that students take an elective in Pediatrics at another institution?

No - only if there is a specific institution that the student is interested in, not a general recommendation.

Pediatric Interest Group (PIG) Please see PIG site

Advisors Assigned by Department Departmental Career Advising Website

Yeshttp://www.washington.edu/medicine/pediatrics/students/current/career

Student Interest Group (S.I.G.) S.I.G. Contact Information

Page 48: Career Advisors FAQ List - education.uwmedicine.org...Career Advisors FAQ List For questions regarding career counseling, please contact SOM Career Advising at medadv@uw.edu Updated

51

5 4 3 2 1Highly Important Less Important

●●

●●

●●

●●●

●Participating in community service

Doing a RUOP or other experience in an underserved communityElection to Gold Humanism Honor Society (GHHS) ●

GHIP or other international experienceHaving an interest in practicing in a rural or underserved community

Having a Master's Degree (eg.MPH)Being bilingual

Having a Step I USMLE score above 235 ●Election to AOA ●

Having a published abstract or poster presentationHaving at least one published paper in a peer reviewed journal

3 letters of recommendation from physicians in this specialty ●

Express a strong interest in academic medicine

1 letter of reference from someone in this specialty1 letter of recommendation from a department chair in this specialty (only if program requires it = rare)

Doing an away rotation in this specialtyDoing a summer research project in this specialty

Doing a preceptorship in this specialty

Doing a UWSOM Explore & Focus Sub-I in this specialty

Achieving "Honors" in this specialty's clerkship rotation

If a student is strongly interested in Pediatrics, please rate how important each of the following activities and performance factors are for a successful residency match.

Achieving 4 or more "Honors" in required clerkship rotations ●

Page 49: Career Advisors FAQ List - education.uwmedicine.org...Career Advisors FAQ List For questions regarding career counseling, please contact SOM Career Advising at medadv@uw.edu Updated

52

Length of Residency Training

3 years

Rate Level of Competitiveness of Specialty

Medium to Low

Mean # of Programs Applied to in this Specialty

32.1

Return to Table of Contents

At what point in the interview season should a student become concerned about their application to Pediatrics residency programs?

If the student has not received the recommended number of interviews by mid-October, they should reach out for help.

If a student is concerned about their application status, what should they do?

Contact Michelle Terry ([email protected])

Residency Application Process - Frequently Asked QuestionsHow many programs should a student apply to in order to be successful in matching in Pediatrics?

Should interview at 12; the number he/she needs to acheive this goal depends on the program choices and the student's academic record.

Recognizing that it can vary from year to year, nationally, when is the typical interview season for Pediatrics?

Mid October-Mid January

# of U.S. M.D. Applicants in this Specialty # of Positions Available

3,241 2768

For more information, please visit the AAMC's Careers in Medicine: Pediatrics website.

CategoricalClinical Practice starting salary: $230,000Academic Med. Assistant Prof: $170,000Academic Med. Full Prof: $209,000

Mean USMLE Step 1 Score Mean USMLE Step 2 CK Score

227 243

National Characteristics of Entering Pediatrics ResidentsCategorical vs. Advanced Program Median Salaries

Page 50: Career Advisors FAQ List - education.uwmedicine.org...Career Advisors FAQ List For questions regarding career counseling, please contact SOM Career Advising at medadv@uw.edu Updated

53

Career Advisor

Dr. Jeffrey Friedrich

Departmental Advisors

Dr. Kari Keys ([email protected]); interested students can contact either Dr. Keys or Dr. Friedrich

Do you have any additional recommendations for students who are committed to matching in this specialty?Start early in MS3 year (in the fall) reaching out to us in plastic surgery to set up a rotation and research plan. Plan to do the Plastic Surgery clerkship late in MS3 year or early in MS4 year because in July, outside rotators arrive and the rotation gets very crowded

Please list the most common mistakes students make when they apply to your specialty:

Waiting too late to set up a plan with us.

Are there any electives outside this specialty that you recommend to students interested in matching into Plastic Surgery?

None are necessary, although helpful clerkships include orthopedics, urology, otolaryngology, dermatology

Frequently Asked QuestionsWhich clerkships are required for students to match in Plastic Surgery? Plastic surgery clerkship at UW

Which UWSOM Sub-Is help students prepare for residency in Plastic Surgery?

Plastic Surgery.

Which Advanced Patient Care clerkships help students prepare for residency in Plastic Surgery? Orthopedics, Dermatology, Otolaryngology, Anesthesia, Medical Consultation.

Plastic SurgeryPreferred Method of Contact Contact Information

Email [email protected]

Do you recommend that students take an elective in Plastic Surgery at another institution?

Yes, 1-2 away electives.

N/A N/A

Advisors Assigned by Department Departmental Career Advising Website

No N/A

Student Interest Group (S.I.G.) S.I.G. Contact Information

Page 51: Career Advisors FAQ List - education.uwmedicine.org...Career Advisors FAQ List For questions regarding career counseling, please contact SOM Career Advising at medadv@uw.edu Updated

54

5 4 3 2 1Highly Important Less Important

●●

●●

●●●

Doing a RUOP or other experience in an underserved communityElection to Gold Humanism Honor Society (GHHS) ●Participating in community service ●

GHIP or other international experience ●Having an interest in practicing in a rural or underserved community

Having a Master's Degree (eg.MPH) ●Being bilingual ●

Having a Step I USMLE score above 235Election to AOA

Having a published abstract or poster presentationHaving at least one published paper in a peer reviewed journal

3 letters of recommendation from physicians in this specialty

Express a strong interest in academic medicine

1 letter of reference from someone in this specialty1 letter of recommendation from a department chair in this specialty

Doing an away rotation in this specialtyDoing a summer research project in this specialty

Doing a preceptorship in this specialty ●Doing a UWSOM Explore & Focus Sub-I in this specialty

Achieving "Honors" in this specialty's clerkship rotation

If a student is strongly interested in Plastic Surgery, please rate how important each of the following activities and performance factors are for a successful residency match.

Achieving 4 or more "Honors" in required clerkship rotations

Page 52: Career Advisors FAQ List - education.uwmedicine.org...Career Advisors FAQ List For questions regarding career counseling, please contact SOM Career Advising at medadv@uw.edu Updated

55

Length of Residency Training

6 years

Rate Level of Competitiveness of Specialty

High

Mean # of Programs Applied to in this Specialty

36.5 (Plastic Surgery - Integrated)

# of U.S. M.D. Applicants in this Specialty # of Positions Available

384 168 (integrated programs only)

For more information, please visit the AAMC's Careers in Medicine: Plastic Surgery website.

Categorical Academic Med. Assistant Prof: $371,000Academic Med. Full Prof: $532,000

Mean USMLE Step 1 Score Mean USMLE Step 2 CK Score

249 254

Return to Table of Contents

At what point in the interview season should a student become concerned about their application to Plastic Surgery residency programs?Late November

At what point in the interview season should a student become concerned about their application to Plastic Surgery residency programs?

Contact Dr. Friedrich, our program director

Residency Application Process - Frequently Asked QuestionsHow many programs should a student apply to in order to be successful in matching in Plastic Surgery?30-40

Recognizing that it can vary from year to year, nationally, when is the typical interview season for Plastic Surgery?

December - January

Additional information:

Drs. Friedrich and Keys welcome the chance to meet with any UW students interested in plastic surgery.

National Characteristics of Entering Plastic Surgery ResidentsCategorical vs. Advanced Program Median Salary

Page 53: Career Advisors FAQ List - education.uwmedicine.org...Career Advisors FAQ List For questions regarding career counseling, please contact SOM Career Advising at medadv@uw.edu Updated

56

Career Advisor

Dr. Mindy Loveless and Dr. Ny-Ying Lam

Departmental Advisors

Drs. Juan Asanza, Ny-Ying Lam, Brian Liem, Denise Li-Lue, Jelena Svircev, Mindy Loveless, Shawn Song, Kelly Pham

Physical Medicine and RehabilitationPreferred Method of Contact Contact Information

Email [email protected]; [email protected]

Do you recommend that students take an elective in PM&R at another institution?

No, it can be an added expense without much benefit unless the student really wants to match there.

Physical Medicine and Rehabilitation Interest Group (PMR)[email protected]

[email protected]

Advisors Assigned by Department Departmental Career Advising Website

Yes

Student Interest Group (S.I.G.) S.I.G. Contact Information

No website

Which UWSOM Sub-Is help students prepare for residency in PM&R?

Ward or Hospital Medicine, Family Medicine, Neurocritical Care Medicine, Advanced Pediatric Clerkship (for those interested in peds rehab).

Which Advanced Patient Care clerkships help students prepare for residency in PM&R?

Are there any electives outside this specialty that you recommend to students interested in matching into PM&R?

None specific for match. Utilize clerkship experiences to gain skills/knowledge useful in PM&R practice such as emergency medicine, ENT, neurology, neurosurgery, ophthalmology, orthopedics, pain, palliative care, radiology, rheumatology, sports medicine, trauma services, urology.

Frequently Asked QuestionsWhich clerkships are required for students to match in PM&R?

At least one PM&R rotation with exposure including inpatient rehab and outpatient clinic

Any Rehab APC, Any Pain Medicine APC in Anesthesia, Sports Medicine (Ortho), Advanced Inpatient or Outpatient Geriatrics (IM Palliative Care).

Page 54: Career Advisors FAQ List - education.uwmedicine.org...Career Advisors FAQ List For questions regarding career counseling, please contact SOM Career Advising at medadv@uw.edu Updated

57

Do you have any additional recommendations for students who are committed to matching in this specialty?(1) Early PM&R exposure is essential to have enough time for core experiences that will support a competitive application to top-tier programs. Complete a UW PM&R Preceptorship and join the UW interest group. Also consider the RIC Externship or AAP RREMS program.(2) Join the AAPM&R - it's free!(3) Gain deeper experience with persons with disabilities. Many students will do this through volunteering.(4) Obtain at least one letter of recommendation from a UW Rehabilitation Medicine core faculty member based in Seattle.(5) If interested in a competitive program, complete a research project or publication in a topic relevant to PM&R.(6) Complete a strong preliminary internal medicine year. A weak internship will leave trainees poorly prepared to care for PM&R patients in residency.

Please list the most common mistakes students make when they apply to your specialty:Lack of knowledge about PM&RNarrow interest in only one aspect of PM&RNot doing enough core rotations in tertiary care centers/hospitals with resident teamsNot explaining problems in the academic record or USMLE scores in the personal statementUnable to explain choice of PM&R as a field over related fieldsTip: In the personal statement, avoid claiming deep understanding of disability based on a personal minor sports injury

Page 55: Career Advisors FAQ List - education.uwmedicine.org...Career Advisors FAQ List For questions regarding career counseling, please contact SOM Career Advising at medadv@uw.edu Updated

58

5 4 3 2 1Highly Important Less Important

●●

●●

If a student is strongly interested in Rehabilitation Medicine, please rate how important each of the following activities and performance factors are for a successful residency match.

Achieving 4 or more "Honors" in required clerkship rotations ●

Achieving "Honors" in this specialty's clerkship rotation

Doing an away rotation in this specialtyDoing a summer research project in this specialty ●

Doing a preceptorship in this specialty

Doing a UWSOM Explore & Focus Sub-I in this specialty

3 letters of recommendation from physicians in this specialty ●

Express a strong interest in academic medicine ●

1 letter of reference from someone in this specialty1 letter of recommendation from a department chair in this specialty ●

Having a Step I USMLE score above 235Election to AOA

Having a published abstract or poster presentation ●Having at least one published paper in a peer reviewed journal

GHIP or other international experience ●Having an interest in practicing in a rural or underserved community ●

Having a Master's Degree (eg.MPH) ●Being bilingual ●

Participating in community service ●

Doing a RUOP or other experience in an underserved community ●Election to Gold Humanism Honor Society (GHHS) ●

Page 56: Career Advisors FAQ List - education.uwmedicine.org...Career Advisors FAQ List For questions regarding career counseling, please contact SOM Career Advising at medadv@uw.edu Updated

59

Length of Residency Training

4 years (including the PGY-1 year)

Rate Level of Competitiveness of Specialty

Medium

Mean # of Programs Applied to in this Specialty

41.9

National Characteristics of Entering Rehabilitation Medicine ResidentsCategorical vs. Advanced Program Median Salaries

# of U.S. M.D. Applicants in this Specialty # of Positions Available

639 133

For more information, please visit the AAMC's Careers in Medicine: Physical Medicine & Rehabilitation website.

Both availableAcademic Med. Assistant Prof: $220,000Academic Med. Full Prof: $250,000

Mean USMLE Step 1 Score Mean USMLE Step 2 CK Score

225 239

Return to Table of Contents

At what point in the interview season should a student become concerned about their application to PM&R residency programs?

Less than 5 PM&R invites within one week of release of the MSPE. Also be mindful of invites for preliminary year programs.

If a student is concerned about their application status, what should they do?

Start by contacting PM&R advisors

Residency Application Process - Frequently Asked QuestionsHow many programs should a student apply to in order to be successful in matching in PM&R?

At least 15, more if any weaknesses in grades, class rank or USMLE scores

Recognizing that it can vary from year to year, nationally, when is the typical interview season for PM&R?

October - December

Page 57: Career Advisors FAQ List - education.uwmedicine.org...Career Advisors FAQ List For questions regarding career counseling, please contact SOM Career Advising at medadv@uw.edu Updated

60

Career Advisor

Dr. Anna Borisovskaya

Departmental AdvisorsDrs. D. Cowley, L. Laplante, J. Markman, C. Varley, S. Romm, J. Buchholz, S. Simmons

Which UWSOM Sub-Is help students prepare for residency in Psychiatry?

Many options in Psychiatry: Ward or Hospital Medicine, Hospitalist and Oncology Subinternship (MEDECK 684).

Which Advanced Patient Care clerkships help students prepare for residency in Psychiatry? Many options in Psychiatry: Dermatology, Opioid Use Disorder (conj. Med/Psych).

Frequently Asked QuestionsWhich clerkships are required for students to match in Psychiatry? Electives in Psychiatry are not required, however they are encouraged. Psychiatry is becoming more competitive as a specialty, which means that having no electives in the 4th year looks odd. Also, doing a 4 week elective early in the year is particularly useful for those who got P or HP in the 3rd year rotation, demonstrating improvement and growth if you get a better grade. There are several excellent rotations offered by our department, and away rotations are also an option. The ones of most interest to 4th year medical students are Consult-Liaison Psychiatry (in any of the teaching hospitals), Psychiatry Emergency Services at HMC, Outpatient Psychiatry at the VA, Sub-I at the VA (on inpatient unit), as well as Geriatric Psychiatry, Child/Adolescent Psychiatry, Detox and Rehab Prog. for Alcoholism and Drug Abuse (Addiction Psychiatry), Forensic Psychiatry.

Do you recommend that students take an elective in Psychiatry at another institution?

No, unless a student absolutely needs to match to that institution and doesn't have a strong application otherwise.

Are there any electives outside this specialty that you recommend to students interested in matching into Psychiatry?

Endocrinology, Cardiology, Dermatology.

PsychiatryPreferred Method of Contact Contact Information

Email [email protected]

Psychiatry/Neurology Interest Group (PNIG) Dr. Eric Kraus - [email protected]

Advisors Assigned by Department Departmental Career Advising Website

Yes http://depts.washington.edu/psyclerk/career.html

Student Interest Group (S.I.G.) S.I.G. Contact Information

Page 58: Career Advisors FAQ List - education.uwmedicine.org...Career Advisors FAQ List For questions regarding career counseling, please contact SOM Career Advising at medadv@uw.edu Updated

61

1. Not emphasizing strongly enough the reason why they're interested in Psychiatry2. Discussing topics in the personal statement that hadn't been fully cognitively processed, such as intense events in one's life. It helps to show your personal statement to your advisor or mentor to get feedback on it.3. Applying to too many or too few residencies, which is probably common to all specialties. Unless there are red flags in your application (failing a USMLE exam, failing a clerkship), applying to 50 residencies is not indicated. On the other hand, you can't just apply to the UW and hope for the best (both scenarios have happened). Talk to your advisor to get a ballpark recommendation for how many residencies to apply to, as everyone's circumstances will be different.4. Acting bored during the interview day: if you're at all interested in the residency, bring your A-game. If you're not interested, just skip the interview.5. It helps to have some subjects to discuss at hand, if there's a lull in conversation on your interview day - patients that changed your experience in medical school or Psychiatry, particular interests of yours in Psychiatry. Not knowing why you're applying to a specific residency, or what may be the particular strengths of that program is a red flag - you should have done your research.6. Weak, uninformative letters of recommendations from preceptors/attendings who clearly either didn't know the student well or didn't actually enjoy working with the student. When asking for letters of recommendations, get them from people who were particularly enthusiastic about you, and word your request as "will you write me a STRONG letter of recommendation?"

Do you have any additional recommendations for students who are committed to matching in this specialty?

While Psychiatry is a less competitive specialty than most others, Admissions Committees look for candidates who are particularly interested in our specialty, who love to listen to people, who are fascinated with the way that someone's mind works - (normally, differently, pathologically), and who have a therapeutic touch to their interactions with patients. Emphasizing these and other pertinent qualities in your CV and personal statement will make your application stand out.

Please list the most common mistakes students make when they apply to your specialty:

Page 59: Career Advisors FAQ List - education.uwmedicine.org...Career Advisors FAQ List For questions regarding career counseling, please contact SOM Career Advising at medadv@uw.edu Updated

62

5 4 3 2 1Highly Important Less Important

●●

Participating in community service ●

Doing a RUOP or other experience in an underserved communityElection to Gold Humanism Honor Society (GHHS) ●

GHIP or other international experience ●Having an interest in practicing in a rural or underserved community

Having a Master's Degree (eg.MPH) ●Being bilingual ●

Having a Step I USMLE score above 235Election to AOA

Having a published abstract or poster presentation ●Having at least one published paper in a peer reviewed journal ●

3 letters of recommendation from physicians in this specialty ●

Express a strong interest in academic medicine ●

1 letter of reference from someone in this specialty1 letter of recommendation from a department chair in this specialty

Doing an away rotation in this specialty ●Doing a summer research project in this specialty

Doing a preceptorship in this specialty ●Doing a UWSOM Explore & Focus Sub-I in this specialty

If a student is strongly interested in Psychiatry, please rate how important each of the following activities and performance factors are for a successful residency match.

Achieving 4 or more "Honors" in required clerkship rotations ●

Achieving "Honors" in this specialty's clerkship rotation

Page 60: Career Advisors FAQ List - education.uwmedicine.org...Career Advisors FAQ List For questions regarding career counseling, please contact SOM Career Advising at medadv@uw.edu Updated

63

Length of Residency Training

4 years (including the PGY-1 year)

Rate Level of Competitiveness of Specialty

Medium, inching toward high

Mean # of Programs Applied to in this Specialty

41.8

Return to Table of Contents

At what point in the interview season should a student become concerned about their application to Psychiatry residency programs?

Probably if they hadn't received invitations from the majority of the residencies they've applied to by the end of November. But they should be aware that some programs do late invitations.

If a student is concerned about their application status, what should they do?

Contact the residency program in question, and ask if they had sent out invitations to interviews. If they have and you didn't get one, ask if they are still reviewing the applications. Express interest in being invited for the interview, because this program is of particular importance to you because of X, Y, Z. If it is vital that you need to go to this program for an interview, please contact me for further advice.

Residency Application Process - Frequently Asked QuestionsHow many programs should a student apply to in order to be successful in matching in Psychiatry?

This varies; depends on student's competitiveness. Highly competitive students may apply to 15-20 programs, less competitive students should apply to higher numbers. Discuss with your advisor. Always apply to a variety of programs in terms of their competitiveness level, as though you were investing in stocks (don't put all your eggs in one type of basket). Apply to some safe programs/community based programs just in case.

Recognizing that it can vary from year to year, nationally, when is the typical interview season for Psychiatry?End of October through end of January (beginning of February in rare cases).

# of U.S. M.D. Applicants in this Specialty # of Positions Available

2,205 1,556

For more information, please visit the AAMC's Careers in Medicine: Psychiatry website.

Categorical Clinical Practice salary: $261,000Academic Med. Assistant Prof: $210,000Academic Med. Full Prof: $242,000

Mean USMLE Step 1 Score Mean USMLE Step 2 CK Score

226 239

National Characteristics of Entering Psychiatry ResidentsCategorical vs. Advanced Program Median Salaries

Page 61: Career Advisors FAQ List - education.uwmedicine.org...Career Advisors FAQ List For questions regarding career counseling, please contact SOM Career Advising at medadv@uw.edu Updated

64

Career Advisor

Dr. Ralph Ermoian

Departmental Advisors

N/A

Radiation OncologyPreferred Method of Contact Contact Information

Email [email protected]

Do you recommend that students take an elective in Radiation Oncology at another institution?

Often students complete 1-2 away rotations, but that is not required. The away rotations can serve many functions including confirming interest in the specialty, building fund of knowledge, exploring whether a particular residency program is a good fit, and creating a favorable impression with a program with which one wants to match. Of note, the latter is a two edge sword; not doing as well as one hopes on an away rotation can make it harder to match to that program.

N/A N/A

Advisors Assigned by Department Departmental Career Advising Website

N/A N/A

Student Interest Group (S.I.G.) S.I.G. Contact Information

Are there any electives outside this specialty that you recommend to students interested in matching into Radiation Oncology?

Medical Oncology; Radiology; Pathology. None of these are necessary but can be helpful.

Frequently Asked QuestionsWhich clerkships are required for students to match in Radiation Oncology?

Radiation Oncology

Which UWSOM Sub-Is help students prepare for residency in Radiation Oncology?

Hospital Medicine/Oncology Subinternship, Oncology Subinternship.

Which Advanced Patient Care clerkships help students prepare for residency in Radiation Oncology?

Palliative Care.

Page 62: Career Advisors FAQ List - education.uwmedicine.org...Career Advisors FAQ List For questions regarding career counseling, please contact SOM Career Advising at medadv@uw.edu Updated

65

Do you have any additional recommendations for students who are committed to matching in this specialty?Most residents expect applicants to have done some research during medical school, ideally with at least one product of research in radiation oncology. Students interested in this specialty are well served to try to work on a project fairly early; however, it is possible to match to radiation oncology residency programs without research in the specialty.

Please list the most common mistakes students make when they apply to your specialty:

1) not applying to enough programs, 2) when interviewing, not preparing for the interview by learning about the program. 3) assuming letters of recommendation have to come from radiation oncologists--at least 1-2 can, but letters from attendings in other specialties who can writing glowing remarks are welcome

Page 63: Career Advisors FAQ List - education.uwmedicine.org...Career Advisors FAQ List For questions regarding career counseling, please contact SOM Career Advising at medadv@uw.edu Updated

66

5 4 3 2 1Highly Important Less Important

●●

●●

●●

●●●

If a student is strongly interested in Radiation Oncology, please rate how important each of the following activities and performance factors are for a successful residency match.

Achieving 4 or more "Honors" in required clerkship rotations

Achieving "Honors" in this specialty's clerkship rotation

Doing an away rotation in this specialty ●Doing a summer research project in this specialty ●

Doing a preceptorship in this specialty

Doing a UWSOM Explore & Focus Sub-I in this specialty

3 letters of recommendation from physicians in this specialty

Express a strong interest in academic medicine

1 letter of reference from someone in this specialty1 letter of recommendation from a department chair in this specialty

Having a Step I USMLE score above 235Election to AOA ●

Having a published abstract or poster presentationHaving at least one published paper in a peer reviewed journal

GHIP or other international experienceHaving an interest in practicing in a rural or underserved community ●

Having a Master's Degree (eg.MPH)Being bilingual

Participating in community service ●

Doing a RUOP or other experience in an underserved communityElection to Gold Humanism Honor Society (GHHS)

Page 64: Career Advisors FAQ List - education.uwmedicine.org...Career Advisors FAQ List For questions regarding career counseling, please contact SOM Career Advising at medadv@uw.edu Updated

67

Length of Residency Training

5 years (including the PGY-1 year)

Rate Level of Competitiveness of Specialty

Medium

Mean # of Programs Applied to in this Specialty

38.1

National Characteristics of Entering Radiation Oncology ResidentsCategorical vs. Advanced Program Median Salaries

# of U.S. M.D. Applicants in this Specialty # of Positions Available

366 16

For more information, please visit the AAMC's Careers in Medicine: Radiation Oncology website.

Both AvailableAcademic Med. Assistant Prof: $380,000Academic Med. Full Prof: $465,000

Mean USMLE Step 1 Score Mean USMLE Step 2 CK Score

247 253

Return to Table of Contents

At what point in the interview season should a student become concerned about their application to Radiation Oncology residency programs?

Fewer than 5 interviews by mid-November.

If a student is concerned about their application status, what should they do?

Contact the department advisor and radiation oncology letter writers.

Residency Application Process - Frequently Asked QuestionsHow many programs should a student apply to in order to be successful in matching in Radiation Oncology?

60-70+

Recognizing that it can vary from year to year, nationally, when is the typical interview season for Radiation Oncology?

Mid-November through January

Page 65: Career Advisors FAQ List - education.uwmedicine.org...Career Advisors FAQ List For questions regarding career counseling, please contact SOM Career Advising at medadv@uw.edu Updated

68

Career AdvisorDrs. Jonathan Medverd (Primary contact) & Gautham Reddy

Departmental AdvisorsTrixie Rombouts: [email protected] (Scheduling Administrator)

Advisors Assigned by Department Departmental Career Advising Website

No N/A

Student Interest Group (S.I.G.) S.I.G. Contact Information

RadiologyPreferred Method of Contact Contact InformationEmail Dr. Medverd at [email protected] and cc Dr. Reddy at [email protected]

[email protected]@uw.edu

Diagnostic and Interventional Radiology Interest Group (DIRIG)Dr. Jonathan Medverd

[email protected]

Do you recommend that students take an elective in Radiology at another institution?DR: Only if you specifically want to go to a particular program and if you are a competitive applicant for that program. Only 5-10% of DR applicants do away electives.IR: Yes,one to two rotations of 4 weeks each. Choose programs at which you would be a competitive applicant. Please seek advice about programs that are appropriate for you and arrange the electives as soon as you can.

Are there any electives outside this specialty that you recommend to students interested in matching into Radiology?

No

Which UWSOM Sub-Is help students prepare for residency in Radiology? Ward or Hospital Medicine, Surgery.

Which Advanced Patient Care clerkships help students prepare for residency in Radiology?

No specific recommendations - choose based on your interests.

Frequently Asked QuestionsWhich clerkships are required for students to match in Radiology?

Applicants to Diagnostic Radiology (DR): Radiology 695, "Radiology Elective Clerkship," 4 weeks in DR in Seattle.Applicants to Interventional Radiology (IR): Radiology 695, "Radiology Elective Clerkship," 4 weeks in IR, plus Radiology 694 "Advanced Radiology Clerkship," 2-4 weeks in DR. The IR rotation must be arranged well in advance. Plan to take a couple of four-week away electives in IR.

Page 66: Career Advisors FAQ List - education.uwmedicine.org...Career Advisors FAQ List For questions regarding career counseling, please contact SOM Career Advising at medadv@uw.edu Updated

69

Please list the most common mistakes students make when they apply to your specialty:

DR:Obtaining all or most letters of recommendation from radiologists.Not applying to enough programs or applying to programs for which you are not competitive.Not completing the application by September 15.In the personal statement, discussing "solving puzzles" or neglecting to mention what you can add to a residency program and your long-term goals. Not mentioning an interest in patient care.Not studying for Step 2. The Step 2 score is not required for the DR application, but if you take it, you don't want the score to be substantially lower than Step 1.Not reaching out to your career advisor multiple times during the application, interview, and ranking process.

IR:Taking only 2 weeks of IR clerkship or not taking a DR clerkship.Applying only to IR rather than dual applying to IR and DR.Obtaining all or most letters of recommendation from interventional or diagnostic radiologists.Not showing enthusiasm and interest during both the IR and DR clerkships.Not applying to enough programs or applying to programs for which you are not competitive.Not completing the application by September 15.In the personal statement, discussing "solving puzzles" or neglecting to mention what you can add to a residency program and your long-term goals. Not mentioning an interest in patient care.Overemphasizing your sole interest in IR (at the expense of DR) during your interviews. The IR residency includes three years of DR, and you will have DR interviewers who want to make sure that you will be engaged during those three years.Not studying for Step 2. The Step 2 score is not required for the IR application, but if you take it, you don't want the score to be substantially lower than Step 1.Not reaching out to your career advisor multiple times during the application, interview, and ranking process.

Do you have any additional recommendations for students who are committed to matching in this specialty?

DR applicants: It is best to take the radiology clerkship no later than Summer B. We will write a departmental letter of recommendation for you, and you can ask specific faculty members to contribute to the letter. You will need two letters from clinical clerkships, preferably including one from medicine or surgery. A letter from a research mentor is acceptable as a fourth letter.

IR applicants: Take the IR clerkship no later than Summer B and DR no later than Summer C. You must obtain a letter of recommendation from an IR faculty member at UW. A surgery letter is strongly recommended. You should consider including your DR letter in the IR application, and you should obtain a fourth letter from a required clinical clerkship. You should dual apply in IR and DR, as IR is extremely competitive.

Page 67: Career Advisors FAQ List - education.uwmedicine.org...Career Advisors FAQ List For questions regarding career counseling, please contact SOM Career Advising at medadv@uw.edu Updated

70

5 4 3 2 1Highly Important Less Important

●●●

●●●

● ●

Doing an away rotation in this specialty ●

Doing a preceptorship in this specialty ●

If a student is strongly interested in Radiology, please rate how important each of the following activities and performance factors are for a successful residency match.

Achieving 4 or more "Honors" in required clerkship rotations ●

Achieving "Honors" in this specialty's clerkship rotation

Doing a summer research project in this specialty

Doing a UWSOM Explore & Focus Sub-I in this specialty ●

3 letters of recommendation from physicians in this specialty ●

Express a strong interest in academic medicine ●

1 letter of reference from someone in this specialty1 letter of recommendation from a department chair in this specialty ●

Having a Step I USMLE score above 235Election to AOA

Having a published abstract or poster presentationHaving at least one published paper in a peer reviewed journal

GHIP or other international experienceHaving an interest in practicing in a rural or underserved community

Having a Master's Degree (eg.MPH)Being bilingual

Participating in community service

Doing a RUOP or other experience in an underserved communityElection to Gold Humanism Honor Society (GHHS)

Page 68: Career Advisors FAQ List - education.uwmedicine.org...Career Advisors FAQ List For questions regarding career counseling, please contact SOM Career Advising at medadv@uw.edu Updated

71

Length of Residency Training

DR: 5 years (including the PGY-1 year) or IR: 6 years (including the PGY-1 year)

Rate Level of Competitiveness of Specialty

High

Mean # of Programs Applied to in this Specialty

DR: 45 IR: 39 (2019 app. yr)

National Characteristics of Entering Radiology ResidentsCategorical vs. Advanced Program Median Salaries

# of U.S. M.D. Applicants in this Specialty # of Positions Available

DR: 925 IR: 237 (2019 app. yr) DR: 965 IR: 113 (2019 app. yr)

For more information, please visit the AAMC's Careers in Medicine: Radiology-Diagnostic website.

Advanced (most programs) or Categorical (some programs)

Clinical Practice salary: DR $419,000 Academic Med. Assistant Prof: $362,000 medianAcademic Med. Full Prof: $433,000 median

Mean USMLE Step 1 Score Mean USMLE Step 2 CK Score

DR: 237, IR: 241 (2019 app. yr) DR: 247, IR: 250 (2019 app. yr)

Return to Table of Contents

At what point in the interview season should a student become concerned about their application to Radiology residency programs?

Most students receive at least 3-5 interview offers by October 15. Students with Step 1 scores under 230 might receive fewer offers, so don't panic, but contact your career advisor if you have less than three by mid October.

If a student is concerned about their application status, what should they do?Contact Radiology DCA to discuss the desirability of applying to more programs and/or contacting residency program coordinators by email.

Residency Application Process - Frequently Asked QuestionsHow many programs should a student apply to in order to be successful in matching in Radiology?

For DR: 40-50. For IR: 20-40 plus 20-50 in DR

Recognizing that it can vary from year to year, nationally, when is the typical interview season for Radiology?There is a wide range, from October to January. You might want to take time off in late October/early November and then again in January. Few interviews are held during the week of Thanksgiving and the week immediately after (RSNA week), and none are held during the winter holiday break.Programs in the Midwest and New England tend to interview earlier, before winter weather sets in.Most programs (other than UW) on the West Coast schedule the majority of their interviews in December and January.

Page 69: Career Advisors FAQ List - education.uwmedicine.org...Career Advisors FAQ List For questions regarding career counseling, please contact SOM Career Advising at medadv@uw.edu Updated

72

Career Advisor

Drs. Kristine Calhoun and Roger Tatum

Departmental AdvisorsDrs. Sam Mandell, Nick Cetrulo, Kathleen Berfield, and Saurabh Khandelwal

SurgeryPreferred Method of Contact Contact Information

via coordinator Laura Yale [email protected]

Do you recommend that students take an elective in Surgery at another institution?

Not unless the student has Step 1 scores significantly below average, an overall weak application, or special circumstances such as a particular interest in one program or city over all others, or for those couples matching where doing an away elective by both members of the couple in the same city may increase the chances of a successful couples match.

Surgery Interest GroupDr. Kathleen Berfield

[email protected]

Advisors Assigned by Department Departmental Career Advising Website

No http://depts.washington.edu/surgstus/career.html

Student Interest Group (S.I.G.) S.I.G. Contact Information

Which UWSOM Sub-Is help students prepare for residency in Surgery?

Surgery Subinternship: General Surgery--SURG 664 (VAMC) or SURG 688 (UWMC), or Vascular Surgery (HMC)--SURG 695; can also do Transplant or CT Surgery at UWMC as alternatives.

Which Advanced Patient Care clerkships help students prepare for residency in Surgery?

Are there any electives outside this specialty that you recommend to students interested in matching into Surgery? Radiology, Anesthesiology, Gastroenterology, Cardiology, Clnical Respiratory Disease (Pulmonary), Subinternship in Critical Care Medicine (Medical ICU).

Frequently Asked QuestionsWhich clerkships are required for students to match in Surgery? Sub-internship (Department of Surgery sub-I preferred by many programs, but Internal Medicine may also work in some cases).

Second sub-I in Surgical ICU or Trauma ICU can fulfill APC requirement - see specific recommendations below. Advanced Inpatient Geriatrics or Advanced Outpatient Geriatrics (Medicine), Palliative Care or Medical Consultation might also be helpful for career preparation.

Page 70: Career Advisors FAQ List - education.uwmedicine.org...Career Advisors FAQ List For questions regarding career counseling, please contact SOM Career Advising at medadv@uw.edu Updated

73

Do you have any additional recommendations for students who are committed to matching in this specialty?

Seek advice sooner rather than later, ideally near the beginning of 3rd year or sooner if considering General Surgery; it is perfectly acceptable to seek an advisor in Surgery when trying to choose between Surgery and another specialty.

Please list the most common mistakes students make when they apply to your specialty:

Not applying to enough programs. Not applying to the appropriate range of programs for relative competitiveness. Not seeking out advice early enough. Not doing enough interviews (recommend minimum of 8 if possible). Not dual applying of record is questionable (Step 1 score under 230).

Page 71: Career Advisors FAQ List - education.uwmedicine.org...Career Advisors FAQ List For questions regarding career counseling, please contact SOM Career Advising at medadv@uw.edu Updated

74

5 4 3 2 1Highly Important Less Important

●●

●●

●●

● ●

If a student is strongly interested in Surgery, please rate how important each of the following activities and performance factors are for a successful residency match.

Achieving 4 or more "Honors" in required clerkship rotations

Achieving "Honors" in this specialty's clerkship rotation

Doing an away rotation in this specialtyDoing a summer research project in this specialty ●

Doing a preceptorship in this specialty ●Doing a UWSOM Explore & Focus Sub-I in this specialty

3 letters of recommendation from physicians in this specialty

Express a strong interest in academic medicine ●

1 letter of reference from someone in this specialty1 letter of recommendation from a department chair in this specialty

Having a Step I USMLE score above 235Election to AOA

Having a published abstract or poster presentation ●Having at least one published paper in a peer reviewed journal ●

GHIP or other international experienceHaving an interest in practicing in a rural or underserved community ●

Having a Master's Degree (eg.MPH) ●Being bilingual

Participating in community service

Doing a RUOP or other experience in an underserved community ●Election to Gold Humanism Honor Society (GHHS)

Page 72: Career Advisors FAQ List - education.uwmedicine.org...Career Advisors FAQ List For questions regarding career counseling, please contact SOM Career Advising at medadv@uw.edu Updated

75

Length of Residency Training

5 years

Rate Level of Competitiveness of Specialty

High

Mean # of Programs Applied to in this Specialty

39.7

National Characteristics of Entering Surgery ResidentsCategorical vs. Advanced Program Median Salaries

# of U.S. M.D. Applicants in this Specialty # of Positions Available

3,963 1,319

For more information, please visit the AAMC's Careers in Medicine: Surgery website.

Categorical with specialtiesAcademic Med. Assistant Prof: $331,000Academic Med. Full Prof: $424,000

Mean USMLE Step 1 Score Mean USMLE Step 2 CK Score

236 248

Return to Table of Contents

At what point in the interview season should a student become concerned about their application to Surgery residency programs?

If they have not received at least 4-5 interviews by 4 weeks or so after the application is submitted (mid-October).

If a student is concerned about their application status, what should they do?

Contact advisor, and email programs to ensure that the application that they have received is complete.

Residency Application Process - Frequently Asked QuestionsHow many programs should a student apply to in order to be successful in matching in Surgery?

30-40 if a competitive applicant (step 1 score at or above application mean of 235, honors in half or more of clerkships), more if less competitive.

Recognizing that it can vary from year to year, nationally, when is the typical interview season for Surgery?

Earliest is late October (only a few programs interview this early), most are between late November and the 3rd week in January.

Page 73: Career Advisors FAQ List - education.uwmedicine.org...Career Advisors FAQ List For questions regarding career counseling, please contact SOM Career Advising at medadv@uw.edu Updated

76

Career Advisor

Dr. Judith Hagedorn

Departmental AdvisorsDr. Mathew Sorensen (Program Director)Dr. John Gore (Research Liaison)Jessica Green (Program Manager)

UrologyPreferred Method of Contact Contact Information

Email [email protected]

N/A N/A

Advisors Assigned by Department Departmental Career Advising Website

Yes N/A

Student Interest Group (S.I.G.) S.I.G. Contact Information

Do you recommend that students take an elective in Urology at another institution?

While this is not required, this is strongly encouraged; students who need to boost their application may choose to perform multiple externships.

Are there any electives outside this specialty that you recommend to students interested in matching into Urology?

Radiology, Nephrology.

Which UWSOM Sub-Is help students prepare for residency in Urology?

Urology.

Which Advanced Patient Care clerkships help students prepare for residency in Urology? Surgical Intensive Care Unit SubI taken for APC credit, Interventional Radiology. Advanced Inpatient Geriatrics or Advanced Outpatient Geriatrics (Medicine), Palliative Care or Medical Consultation might also be helpful for career preparation.

Frequently Asked QuestionsWhich clerkships are required for students to match in Urology?

Typical third year clerkships. As well as participation in the UW Urology sub-intern elective (685).

Page 74: Career Advisors FAQ List - education.uwmedicine.org...Career Advisors FAQ List For questions regarding career counseling, please contact SOM Career Advising at medadv@uw.edu Updated

77

Do you have any additional recommendations for students who are committed to matching in this specialty?

Early engagement with the Department as Urology is a competitive early match. Successful applicants have strong board scores and clerkship grades as well as some research experience. Early engagement allows completion of meaningful research projects and provides the Department more information for a strong letter of recommendation.

Please list the most common mistakes students make when they apply to your specialty:Deciding late in 3rd year, not allowing time for preparation of in-house and away sub-Is. Honest assessment of academic achievement and scores are necessary to gauge likelihood of matching. If students think they may be interested in Urology, they should reach out to the department early. Sub-intern rotations (at UW and away) tend to fill early and as Urology is an early match, rotations need to be completed by the end of August. Spots tend to fill early .

Page 75: Career Advisors FAQ List - education.uwmedicine.org...Career Advisors FAQ List For questions regarding career counseling, please contact SOM Career Advising at medadv@uw.edu Updated

78

5 4 3 2 1Highly Important Less Important

●●

●●

●●

●●●

Doing a preceptorship in this specialty

Doing a UWSOM Explore & Focus Sub-I in this specialty

If a student is strongly interested in Urology, please rate how important each of the following activities and performance factors are for a successful residency match.

Achieving 4 or more "Honors" in required clerkship rotations

Achieving "Honors" in this specialty's clerkship rotation

1 letter of reference from someone in this specialty1 letter of recommendation from a department chair in this specialty

Doing an away rotation in this specialtyDoing a summer research project in this specialty

Having a published abstract or poster presentationHaving at least one published paper in a peer reviewed journal ●

3 letters of recommendation from physicians in this specialty

Express a strong interest in academic medicine ●

Having a Master's Degree (eg.MPH)Being bilingual

Having a Step I USMLE score above 235Election to AOA

Doing a RUOP or other experience in an underserved community ●Election to Gold Humanism Honor Society (GHHS)

GHIP or other international experienceHaving an interest in practicing in a rural or underserved community ●

Participating in community service ●

Page 76: Career Advisors FAQ List - education.uwmedicine.org...Career Advisors FAQ List For questions regarding career counseling, please contact SOM Career Advising at medadv@uw.edu Updated

79

Length of Residency Training

5 years (including the PGY-1 year)

Rate Level of Competitiveness of Specialty

Typically high, according to the American Urological Association the match rate in 2018 was 78%, which was actually higher than the typical average of 65-70%.

Mean # of Programs Applied to in this Specialty

71

AdvancedClinical Practice salary: $386,000Academic Med. Assistant Prof: $343,000Academic Med. Full Prof: $447,000

Mean USMLE Step 1 Score Mean USMLE Step 2 CK Score

240 244

National Characteristics of Entering Urology ResidentsCategorical vs. Advanced Program Median Salaries

Residency Application Process - Frequently Asked QuestionsHow many programs should a student apply to in order to be successful in matching in Urology?Apply to 30-40 programs, with a target of approximately 10-15 interviews.

Recognizing that it can vary from year to year, nationally, when is the typical interview season for Urology?

October through December. Being on a rotation that would allow absences (or scheduling time off) in this time period is advisable.

# of Applicants in this Specialty # of Positions Available

434 339

For more information, please visit the AAMC's Careers in Medicine: Urology website.

Additional information:

Urology Match is a good resource.

Return to Table of Contents

At what point in the interview season should a student become concerned about their application to Urology residency programs?

Fewer than 10 interview offers by mid-November.

If a student is concerned about their application status, what should they do?

Contact SOM Advisor in Urology and/or UW Urology Program Director.